Você está na página 1de 140

FÍSICA

Conservação da Quantidade de
Movimento e Colisões

SISTEMA DE ENSINO

Livro Eletrônico
FÍSICA
Conservação da Quantidade de Movimento e Colisões
Herico Avohai

Conservação da Quantidade de Movimento e Colisões. . ...................................................4


1. Conservação da Quantidade de Movimento..................................................................4
1.1. Sistema Isolado.........................................................................................................5
2. Colisões..................................................................................................................... 14
2.1. Tipos de Colisões.....................................................................................................17
3. Coeficiente de Restituição (e)....................................................................................29
Questões de Concurso...................................................................................................36
Gabarito........................................................................................................................55
Gabarito Comentado. .....................................................................................................56

O conteúdo deste livro eletrônico é licenciado para DÉBORA LIZ RIBEIRO STELLE - , vedada, por quaisquer meios e a qualquer título,
a sua reprodução, cópia, divulgação ou distribuição, sujeitando-se aos infratores à responsabilização civil e criminal.
www.grancursosonline.com.br 2 de 140
FÍSICA
Conservação da Quantidade de Movimento e Colisões
Herico Avohai

Apresentação

Diga aí, tudo bem?... Espero que não esteja sumido(a)!! Eu e o Bizurado estamos na torcida
pelo seu sucesso.

Já dizia Homer Simpson “Tentar é o primeiro passo rumo ao fracasso”, e ele tem razão,
entretanto para haver sucesso, é preciso dar o primeiro passo e continuar até conquistar o seu
objetivo. Sabemos que as nossas palavras têm poderes, então, as afirmações “Eu posso!” “Eu
quero!” “Eu tomo posse!” “Eu consigo!” “Eu sou!”, devem fazer parte do seu dia a dia.
Nesta aula estudaremos sobre a Conservação da Quantidade de Movimento e as colisões.
Chamo a sua atenção para as questões referentes a pêndulo balístico, pois caem frequen-
temente nas provas.
Tudo certo!?? Bora começar!??

O conteúdo deste livro eletrônico é licenciado para DÉBORA LIZ RIBEIRO STELLE - , vedada, por quaisquer meios e a qualquer título,
a sua reprodução, cópia, divulgação ou distribuição, sujeitando-se aos infratores à responsabilização civil e criminal.
www.grancursosonline.com.br 3 de 140
FÍSICA
Conservação da Quantidade de Movimento e Colisões
Herico Avohai

CONSERVAÇÃO DA QUANTIDADE DE MOVIMENTO E


COLISÕES
1. Conservação da Quantidade de Movimento

Imagine um casal de patinadores,

Imagem 1: https://www.freepik.com/free-photo/cute-couple-ice-arena_4050276.htm

Sobre esse casal de patinadores podem agir forças internas ou externas.


As forças internas são aquelas em que as partículas (patinadores) trocam entre si, note
que se o patinador aplicar uma força na patinadora, sabemos que por consequência da 3ª Lei
de Newton, a patinadora aplicará uma força no patinador (ação e reação). Com isso, podemos
concluir que o Impulso total devido às forças internas é nulo.
As forças externas são aquelas exercidas por um agente externo ao sistema.

O conteúdo deste livro eletrônico é licenciado para DÉBORA LIZ RIBEIRO STELLE - , vedada, por quaisquer meios e a qualquer título,
a sua reprodução, cópia, divulgação ou distribuição, sujeitando-se aos infratores à responsabilização civil e criminal.
www.grancursosonline.com.br 4 de 140
FÍSICA
Conservação da Quantidade de Movimento e Colisões
Herico Avohai

1.1. Sistema Isolado

Consideramos um sistema isolado quando ele possui as seguintes principais característi-


cas:
• a soma das forças externas é nula;
• o Impulso total das forças externas é nulo;
• quando o valor das forças externas for desprezível em comparação com as forças in-
ternas;
• quando a interação das forças externas for um intervalo de tempo muiiiiiitooo pequeno.

Então, se considerarmos o sistema (patinador+patinadora) como isolado, ele estará livre


das ações de agentes externos, logo o impulso total do sistema é nulo.
Imagine que inicialmente os patinadores estão em repouso de mãos dadas e um de frente
para o outro.
Esquematizando o sistema isolado e com o referencial positivo para a direita, temos,

Imagem 2: Patinadores em repouso.

O que acontece nesse momento, antes de qualquer interação entre ele?


Antes da interação entre eles, os patinadores estão em repouso, logo a Quantidade de Mo-
vimento Total do sistema isolado antes é igual a zero, pois eles estão em repouso.

O conteúdo deste livro eletrônico é licenciado para DÉBORA LIZ RIBEIRO STELLE - , vedada, por quaisquer meios e a qualquer título,
a sua reprodução, cópia, divulgação ou distribuição, sujeitando-se aos infratores à responsabilização civil e criminal.
www.grancursosonline.com.br 5 de 140
FÍSICA
Conservação da Quantidade de Movimento e Colisões
Herico Avohai

Agora imagine que um empurra o outro e passam a se movimentar em sentidos opostos

Imagem 3: Patinadores se movimentando após o empurrão.

Observa-se que após o empurrão, cada um dos patinadores se movimenta com velocida-
des diferentes, pois possuem massas diferentes.
Logo, a Quantidade de Movimento Total do sistema isolado após a interação entre eles é
dada por,

Para encontrar o módulo da Quantidade de Movimento depois, temos que realizar a soma
vetorial,

Então, o módulo do vetor Quantidade de Movimento Total depois da interação será:

Voltando ao sistema isolado, já vimos que o Impulso total (I = 0) é igual a zero, portanto
aplicando o Teorema do Impulso e substituindo os valores conhecidos, temos,

O conteúdo deste livro eletrônico é licenciado para DÉBORA LIZ RIBEIRO STELLE - , vedada, por quaisquer meios e a qualquer título,
a sua reprodução, cópia, divulgação ou distribuição, sujeitando-se aos infratores à responsabilização civil e criminal.
www.grancursosonline.com.br 6 de 140
FÍSICA
Conservação da Quantidade de Movimento e Colisões
Herico Avohai

Portanto, “a quantidade de movimento de um sistema isolado é conservada”.

Para o exemplo em questão, temos que,

Fica fácil observar que a menor massa adquire a maior velocidade e a maior massa adquire
a menor velocidade.

Professor, como estava o sistema isolado antes e como está depois da interação das
forças internas?

Porque a partir daí você analisará quem estava em repouso, quem estava em movimento,
quais os sentidos das velocidades e em seguida realizar a soma vetorial e aplicar da conserva-
ção da Quantidade de Movimento. Ok??
Vamos ao primeiro exemplo.

O conteúdo deste livro eletrônico é licenciado para DÉBORA LIZ RIBEIRO STELLE - , vedada, por quaisquer meios e a qualquer título,
a sua reprodução, cópia, divulgação ou distribuição, sujeitando-se aos infratores à responsabilização civil e criminal.
www.grancursosonline.com.br 7 de 140
FÍSICA
Conservação da Quantidade de Movimento e Colisões
Herico Avohai

Exemplo 1: (PUC/VESTIBULAR/2014) Dois patinadores, um de massa 100kg e outro de massa


80kg, estão de mãos dadas em repouso sobre uma pista de gelo, onde o atrito é desprezível.
Eles empurram-se mutuamente e deslizam na mesma direção, porém em sentidos opostos. O
patinador de 100kg adquire uma velocidade de 4m/s. A velocidade relativa de um dos patina-
dores em relação ao outro é, em módulo, igual a:

a) 5 m/s
b) 4 m/s
c) 1 m/s
d) 9 m/s
e) 20 m/s

Letra d.
DADOS
M = 100 kg
m = 80 kg
vM = 4 m/s
Vrelativa =?
Considerando o referencial positivo para direita e o sistema isolado, temos que a Quantidade
de Movimento se conserva.

O conteúdo deste livro eletrônico é licenciado para DÉBORA LIZ RIBEIRO STELLE - , vedada, por quaisquer meios e a qualquer título,
a sua reprodução, cópia, divulgação ou distribuição, sujeitando-se aos infratores à responsabilização civil e criminal.
www.grancursosonline.com.br 8 de 140
FÍSICA
Conservação da Quantidade de Movimento e Colisões
Herico Avohai

ANTES
Os patinadores estão em repouso, logo,

DEPOIS
Os patinadores estão em movimento em sentidos contrários, logo,

Para encontrar o módulo da Quantidade de Movimento depois, temos que realizar a soma vetorial,

Então, o módulo do vetor Quantidade de Movimento depois será:

O conteúdo deste livro eletrônico é licenciado para DÉBORA LIZ RIBEIRO STELLE - , vedada, por quaisquer meios e a qualquer título,
a sua reprodução, cópia, divulgação ou distribuição, sujeitando-se aos infratores à responsabilização civil e criminal.
www.grancursosonline.com.br 9 de 140
FÍSICA
Conservação da Quantidade de Movimento e Colisões
Herico Avohai

Substituindo os valores na equação da conservação da Quantidade de Movimento,

Para terminar, encontraremos a velocidade relativa.


Como os dois patinadores estão se movimentando em sentidos opostos, a velocidade relativa
será a soma dos módulos das velocidades de cada patinador,

Questão 1 (CEBRASPE/VESTIBULAR/2008) Os veículos espaciais são conduzidos por fo-


guetes que se deslocam no espaço aéreo em sentido oposto aos gases expelidos pelos moto-
res. Esses gases são obtidos a partir de combustíveis utilizados pelos foguetes. Esses veículos
atingem altas velocidades para deixarem o campo gravitacional da Terra. Para resistir às ele-
vadas temperaturas verificadas durante a reentrada na atmosfera terrestre, esses veículos são
revestidos por materiais especiais para proteção térmica, que incluem principalmente mate-
riais cerâmicos. Julgue o item:
Considere que um foguete em sua plataforma de lançamento possui uma massa total igual a
5.000 kg. Considere ainda que, no lançamento, este foguete expele instantaneamente 1.000 kg
de gás à velocidade de 3.000 m/s em relação a ele. Nessa situação, considerando-se o princí-

O conteúdo deste livro eletrônico é licenciado para DÉBORA LIZ RIBEIRO STELLE - , vedada, por quaisquer meios e a qualquer título,
a sua reprodução, cópia, divulgação ou distribuição, sujeitando-se aos infratores à responsabilização civil e criminal.
www.grancursosonline.com.br 10 de 140
FÍSICA
Conservação da Quantidade de Movimento e Colisões
Herico Avohai

pio de conservação da quantidade de movimento, a velocidade adquirida pelo foguete, após


ejetar essa massa de gás, é inferior a 700 m/s.

Errado.
DADOS
M = 4000 kg
mexpelida = 1000 kg
vexpelida = 3000 m/s
Vfoguete =?
Note que o examinador considera a massa total do foguete igual a 5000 kg, portanto, sem a
massa do gás, a massa do foguete é 4000 kg.
Considerando o referencial positivo para cima e o sistema isolado, temos que a Quantidade de
Movimento se conserva.

ANTES
O foguete está em repouso,

DEPOIS
O foguete sobe e a massa é expelida no sentido contrário, logo,

O conteúdo deste livro eletrônico é licenciado para DÉBORA LIZ RIBEIRO STELLE - , vedada, por quaisquer meios e a qualquer título,
a sua reprodução, cópia, divulgação ou distribuição, sujeitando-se aos infratores à responsabilização civil e criminal.
www.grancursosonline.com.br 11 de 140
FÍSICA
Conservação da Quantidade de Movimento e Colisões
Herico Avohai

Para encontrar o módulo da Quantidade de Movimento depois, temos que realizar a soma ve-
torial,

Então, o módulo do vetor Quantidade de Movimento depois será:

Substituindo os valores na equação da conservação da Quantidade de Movimento,

O conteúdo deste livro eletrônico é licenciado para DÉBORA LIZ RIBEIRO STELLE - , vedada, por quaisquer meios e a qualquer título,
a sua reprodução, cópia, divulgação ou distribuição, sujeitando-se aos infratores à responsabilização civil e criminal.
www.grancursosonline.com.br 12 de 140
FÍSICA
Conservação da Quantidade de Movimento e Colisões
Herico Avohai

A próxima questão é para você resolver sozinha(o), seguindo os raciocínios das questões
anteriores.

Questão 2 (EXÉRCITO/CADETE/ESPCEX/2011) Um canhão, inicialmente em repouso, de


massa 600 kg, dispara um projétil de massa 3 kg com velocidade horizontal de 800 m/s. Des-
prezando todos os atritos, podemos afirmar que a velocidade de recuo do canhão é de:e
a) 2 m/s
b) 4 m/s
c) 6 m/s
d) 8 m/s
e) 12 m/s

O conteúdo deste livro eletrônico é licenciado para DÉBORA LIZ RIBEIRO STELLE - , vedada, por quaisquer meios e a qualquer título,
a sua reprodução, cópia, divulgação ou distribuição, sujeitando-se aos infratores à responsabilização civil e criminal.
www.grancursosonline.com.br 13 de 140
FÍSICA
Conservação da Quantidade de Movimento e Colisões
Herico Avohai

2. Colisões

Gosto muito desse conteúdo, pois faz parte da rotina de trabalho da Perícia Criminal.

Fotografia 1: Colisão frontal

O conteúdo deste livro eletrônico é licenciado para DÉBORA LIZ RIBEIRO STELLE - , vedada, por quaisquer meios e a qualquer título,
a sua reprodução, cópia, divulgação ou distribuição, sujeitando-se aos infratores à responsabilização civil e criminal.
www.grancursosonline.com.br 14 de 140
FÍSICA
Conservação da Quantidade de Movimento e Colisões
Herico Avohai

Fotografia 2: Colisão perpendicular.

Colisões ou choques são interações entre corpos que podem ser:


• Unidimensionais: são aquelas que ocorrem em uma direção, por exemplo, colisão fron-
tal e colisão traseira;
• Bidimensionais: são aquelas que ocorrem em duas direções, por exemplo, colisão de
veículos em um cruzamento;
• Tridimensionais: são aquelas que ocorrem em três direções, por exemplo, explosões de
objetos.

A principal característica das colisões é que em qualquer uma delas A QUANTIDADE DE


MOVIMENTO SE CONSERVA!!!

O conteúdo deste livro eletrônico é licenciado para DÉBORA LIZ RIBEIRO STELLE - , vedada, por quaisquer meios e a qualquer título,
a sua reprodução, cópia, divulgação ou distribuição, sujeitando-se aos infratores à responsabilização civil e criminal.
www.grancursosonline.com.br 15 de 140
FÍSICA
Conservação da Quantidade de Movimento e Colisões
Herico Avohai

Exemplo 1: Considere que o veículo A de massa mA com velocidade horizontal e para direita
colide com o veículo B de massa mB que está inicialmente em repouso, após a colisão o veícu-
lo A fica parado e o veículo B passa a se movimentar com velocidade horizontal e para direita.
Adote o referencial positivo para a direita.

Vamos considerar o exemplo acima somente para esquematizá-lo e aplicar a equação da


conservação da Quantidade de Movimento.
Portanto, inicialmente,

O vetor quantidade de movimento total antes será a soma dos vetores quantidades de mo-
vimento de A e B.

E o seu módulo será,

É de se esperar isso, pois somente A está se movimentando.


Após a colisão,

Os dois passam a movimentar-se para a direita, portanto a o vetor quantidade de movimen-


to depois será a soma dos vetores quantidades de movimento de A e B.

O conteúdo deste livro eletrônico é licenciado para DÉBORA LIZ RIBEIRO STELLE - , vedada, por quaisquer meios e a qualquer título,
a sua reprodução, cópia, divulgação ou distribuição, sujeitando-se aos infratores à responsabilização civil e criminal.
www.grancursosonline.com.br 16 de 140
FÍSICA
Conservação da Quantidade de Movimento e Colisões
Herico Avohai

Como os vetores quantidades de movimentado estão na mesma direção e no mesmo sen-


tido, o seu módulo será,

Substituindo na equação da conservação a Quantidade de Movimento,

Pronto, essa é a análise que você fará em todas as questões sobre choques ou colisões.

2.1. Tipos de Colisões

Existem três tipos de colisões.

a) Colisão Perfeitamente Elástica. Tem como características a conservação da quantidade

de movimento (como todas né!!) e a conservação da Energia Mecânica, sendo que após a cho-

que os corpos permanecem separados.

ANTES DA COLISÃO

DEPOIS DA COLISÃO

O conteúdo deste livro eletrônico é licenciado para DÉBORA LIZ RIBEIRO STELLE - , vedada, por quaisquer meios e a qualquer título,
a sua reprodução, cópia, divulgação ou distribuição, sujeitando-se aos infratores à responsabilização civil e criminal.
www.grancursosonline.com.br 17 de 140
FÍSICA
Conservação da Quantidade de Movimento e Colisões
Herico Avohai

Muita atenção, aspira!! Falei três características importante para esse tipo de colisão:

• há conservação de Quantidade de Movimento;

• há conservação de Energia;

• após a colisão os corpos ficam separados.

b) Colisão Parcialmente Elástica ou parcialmente inelástica. Tem como característica a

conservação da quantidade de movimento (como todas né!!), porém não há conservação da

Energia Mecânica, sendo que após a choque os corpos permanecem separados.

ANTES DA COLISÃO

DEPOIS DA COLISÃO

Mais uma vez, aspira!! Falei três características importante para esse tipo de colisão:

• há conservação de quantidade de movimento;

• não há conservação de energia;

• após a colisão os corpos ficam separados.

Minha(meu) amiga(o), essas duas colisões são bem parecidas, né? A única diferença é

que na colisão perfeitamente elástica há conservação de Energia e na colisão parcialmente

elástica não há.

O conteúdo deste livro eletrônico é licenciado para DÉBORA LIZ RIBEIRO STELLE - , vedada, por quaisquer meios e a qualquer título,
a sua reprodução, cópia, divulgação ou distribuição, sujeitando-se aos infratores à responsabilização civil e criminal.
www.grancursosonline.com.br 18 de 140
FÍSICA
Conservação da Quantidade de Movimento e Colisões
Herico Avohai

Fazia tempo que você não perguntava, heim, Bizurado!!! A questão deve deixar claro, ou

então ela pode pedir para você diferencias as colisões pelas características dadas, daí é res-

ponder e correr para o abraço!!!!

c) Colisão Perfeitamente Inelástica (anelástica). Tem como característica a conservação

da quantidade de movimento (como todas né!!), porém não há conservação da Energia Mecâ-

nica, sendo que após a choque os corpos permanecem juntos (grudados) e movimentam-se

com a mesma velocidade.


ANTES DA COLISÃO

DEPOIS DA COLISÃO

Por fim, stivado(a)!! Mais três características importante para esse tipo de colisão:
• há conservação de Quantidade de Movimento;

O conteúdo deste livro eletrônico é licenciado para DÉBORA LIZ RIBEIRO STELLE - , vedada, por quaisquer meios e a qualquer título,
a sua reprodução, cópia, divulgação ou distribuição, sujeitando-se aos infratores à responsabilização civil e criminal.
www.grancursosonline.com.br 19 de 140
FÍSICA
Conservação da Quantidade de Movimento e Colisões
Herico Avohai

• não há conservação de Energia;


• após a colisão os corpos ficam juntos (grudados).

Então todas as vezes que o examinador colocar na questão que após o choque os corpos
permanecem juntos ou grudados, estamos trabalhando com colisão perfeitamente inelástica,
ok!!??
Uma tabelinha para ficar mais fácil de lembrar:

CONSERVAÇÃO DA
CONSERVAÇÃO DE CARACTERÍSTICA APÓS
TIPOS DE COLISÃO QUANTIDADE DE
ENERGIA A COLISÃO
MOVIMENTO
PERF. ELÁSTICA SIM SIM SEPARADOS
PARCIAL. ELÁSTICA SIM NÃO SEPARADOS
PERF. INELÁSTICA SIM NÃO JUNTOS

Exemplo 2: (UFSM-RS/VESTIBULAR/2009) Uma flecha de massa 100g, a uma velocidade de


24m/s encontra uma ave, com massa de 900g, livre, em repouso sobre um galho. A ave ferida
mais a flecha passam a ser um único corpo, com velocidade final, em m/s, de
a) zero.
b) 0,6.
c) 1,2.
d) 2,4.
e) 6.

Letra d.
DADOS
mFLECHA = 100 g = 0,1 kg
mAVE = 900 g = 0,9 kg
Antes da colisão

O conteúdo deste livro eletrônico é licenciado para DÉBORA LIZ RIBEIRO STELLE - , vedada, por quaisquer meios e a qualquer título,
a sua reprodução, cópia, divulgação ou distribuição, sujeitando-se aos infratores à responsabilização civil e criminal.
www.grancursosonline.com.br 20 de 140
FÍSICA
Conservação da Quantidade de Movimento e Colisões
Herico Avohai

vFLECHA = 24 m/s
vAVE = 0
Depois da colisão
vCONJUNTO =?
mCONJUNTO = mFLECHA + mAVE = 0,1 + 0,9 = 1 kg
Tadinho do passarinho!! 1 Não faça isso em casa! É só uma questão e nenhum passarinho
foi maltratado!
Brincadeiras à parte, olha só, que beleza!! O enunciado diz que “A ave ferida mais a flecha
passam a ser um único corpo”, ou seja, estamos diante da colisão perfeitamente inelástica.
Esquematizando o problema,

ANTES DA COLISÃO

O vetor quantidade de movimento total antes será a soma dos vetores quantidades de mo-
vimento da flecha e do passarinho.

E o seu módulo será,

O conteúdo deste livro eletrônico é licenciado para DÉBORA LIZ RIBEIRO STELLE - , vedada, por quaisquer meios e a qualquer título,
a sua reprodução, cópia, divulgação ou distribuição, sujeitando-se aos infratores à responsabilização civil e criminal.
www.grancursosonline.com.br 21 de 140
FÍSICA
Conservação da Quantidade de Movimento e Colisões
Herico Avohai

DEPOIS DA COLISÃO

Depois do choque, os dois movimentam-se juntos e para a direita, portanto o vetor quanti-
dade de movimento depois será a o vetor quantidade de movimento da flecha e do passarinho
(juntos).

E em módulo,

Substituindo na equação da conservação a Quantidade de Movimento,

O conteúdo deste livro eletrônico é licenciado para DÉBORA LIZ RIBEIRO STELLE - , vedada, por quaisquer meios e a qualquer título,
a sua reprodução, cópia, divulgação ou distribuição, sujeitando-se aos infratores à responsabilização civil e criminal.
www.grancursosonline.com.br 22 de 140
FÍSICA
Conservação da Quantidade de Movimento e Colisões
Herico Avohai

Não poderia, pois a colisão é perfeitamente inelástica e vimos que uma das características
desse tipo de colisão é que a Energia não é conservada.
Muita atenção na próxima questão!! Os examinadores gostam muito!

Questão 3 (CEBRASPE/PERITO CRIMINAL/POLÍCIA CIENTÍFICA-PE/2016) Em um estan-


de de tiro, um perito, para estimar a velocidade de um projétil de arma de fogo, atirou contra um
pêndulo balístico, conforme ilustrado na figura precedente, e mediu a altura máxima atingida
pelo pêndulo após o choque.
Sabendo-se que esse projétil possui massa de 50 g, que o bloco possui massa de 5 kg, que o
projétil ficou alojado no bloco após o choque, que a altura máxima medida pelo perito foi de 20
cm e que a aceleração da gravidade no local era de 10 m/s2, é correto afirmar que a velocidade
com que o projétil atingiu o bloco foi de

O conteúdo deste livro eletrônico é licenciado para DÉBORA LIZ RIBEIRO STELLE - , vedada, por quaisquer meios e a qualquer título,
a sua reprodução, cópia, divulgação ou distribuição, sujeitando-se aos infratores à responsabilização civil e criminal.
www.grancursosonline.com.br 23 de 140
FÍSICA
Conservação da Quantidade de Movimento e Colisões
Herico Avohai

a) 206 m/s.
b) 208 m/s.
c) 200 m/s.
d) 202 m/s.
e) 204 m/s.

Letra d.
DADOS
mPROJÉTIL = 50 g = 0,05 kg
h = 20 cm = 0,2 m
Mbloco = 4 kg
Futuro(a) Comandante da Marinha Brasileira, eu ouvi um amém???
Se eu pudesse apostar numa questão da prova, seria desse tipo. Os examinadores têm colo-
cado questões sobre pêndulo balístico e depois dessa aula, você não errará mais, promete?
Pelas características do problema, podemos dizer que o choque (ou colisão) é classificado
como perfeitamente inelástico, pois após a colisão, o projétil e o bloco fincam juntos.
Desse modo, não há conservação de energia durante o choque, entretanto há conservação da
quantidade de movimento.

O conteúdo deste livro eletrônico é licenciado para DÉBORA LIZ RIBEIRO STELLE - , vedada, por quaisquer meios e a qualquer título,
a sua reprodução, cópia, divulgação ou distribuição, sujeitando-se aos infratores à responsabilização civil e criminal.
www.grancursosonline.com.br 24 de 140
FÍSICA
Conservação da Quantidade de Movimento e Colisões
Herico Avohai

Todavia, após o choque (depois que o projétil ficou alojado), eu disse APÓS O CHOQUE, pode-
mos conservar a energia mecânica do sistema.
Portanto, primeiramente iremos calcular a velocidade que o conjunto (projétil + bloco) adquire
logo após a colisão e em seguida aplicar a conservação da Quantidade de Movimento para
encontrar a velocidade com que o projétil atingiu o bloco.
Após o impacto,

Para encontrar a velocidade com que o conjunto (bloco + projétil) sai após o impacto, aplicare-
mos a Conservação da Energia Mecânica nos pontos A e B.

No ponto A, considerando o referencial nesse nível, o conjunto possui Energia Cinética diferen-
te de zero e Energia Potencial Gravitacional igual a zero.

No ponto B, o conjunto possui Energia Cinética igual a zero, pois atinge a altura máxima, e Ener-
gia Potencial Gravitacional diferente de zero.

O conteúdo deste livro eletrônico é licenciado para DÉBORA LIZ RIBEIRO STELLE - , vedada, por quaisquer meios e a qualquer título,
a sua reprodução, cópia, divulgação ou distribuição, sujeitando-se aos infratores à responsabilização civil e criminal.
www.grancursosonline.com.br 25 de 140
FÍSICA
Conservação da Quantidade de Movimento e Colisões
Herico Avohai

Igualando as Energias mecânicas e substituindo os valores conhecidos,

Para calcular a velocidade do projétil antes do choque, vamos aplicar a conservação da quan-

tidade de movimento:

ANTES DA COLISÃO

O vetor quantidade de movimento total antes será a soma dos vetores quantidades de movi-

mento do projétil e do bloco.

O bloco está em repouso e o projétil em movimento, logo o módulo da quantidade de movimen-

to total antes, será,

O conteúdo deste livro eletrônico é licenciado para DÉBORA LIZ RIBEIRO STELLE - , vedada, por quaisquer meios e a qualquer título,
a sua reprodução, cópia, divulgação ou distribuição, sujeitando-se aos infratores à responsabilização civil e criminal.
www.grancursosonline.com.br 26 de 140
FÍSICA
Conservação da Quantidade de Movimento e Colisões
Herico Avohai

DEPOIS DA COLISÃO
O vetor quantidade de movimento total depois será a quantidade de movimento do conjunto
(projétil + bloco).

Depois da colisão, o conjunto (projétil+bloco) se movimenta, logo o módulo da quantidade de


movimento total depois, será,

Substituindo na equação da conservação da quantidade de movimento, temos,

Só mais uma!! Trouxe uma questão envolvendo gráfico. A leitura do gráfico é importante
também para a resolução das questões.

Questão 4 (VUNESP/VESTIBULAR/2000) A figura mostra o gráfico das velocidades de


dois carrinhos que se movem sem atrito sobre um mesmo par de trilhos horizontais e retilíne-
os. Em torno do instante 3 segundos, os carrinhos colidem.

O conteúdo deste livro eletrônico é licenciado para DÉBORA LIZ RIBEIRO STELLE - , vedada, por quaisquer meios e a qualquer título,
a sua reprodução, cópia, divulgação ou distribuição, sujeitando-se aos infratores à responsabilização civil e criminal.
www.grancursosonline.com.br 27 de 140
FÍSICA
Conservação da Quantidade de Movimento e Colisões
Herico Avohai

Se as massas dos carrinhos 1 e 2 são, respectivamente, m1 e m2, então:


a) m1 = 3m2
b) 3m1 = m2
c) 3m1 = 5m2
d) 3m1 = 7m2
e) 5m1 = 3m2

Letra e.
Trouxe essa questão para você ter a noção do formato do gráfico de uma colisão.
Iremos interpretar o gráfico, logo em seguida, aplicar a conservação da Quantidade de
Movimento.

Se a colisão ocorreu por volta de 3 s, temos então que:

O conteúdo deste livro eletrônico é licenciado para DÉBORA LIZ RIBEIRO STELLE - , vedada, por quaisquer meios e a qualquer título,
a sua reprodução, cópia, divulgação ou distribuição, sujeitando-se aos infratores à responsabilização civil e criminal.
www.grancursosonline.com.br 28 de 140
FÍSICA
Conservação da Quantidade de Movimento e Colisões
Herico Avohai

• a velocidade inicial do carrinho 1 é – 2 m/s;


• a velocidade inicial do carrinho 2 é + 4 m/s;
• a velocidade final do carrinho 1 é + 3 m/s;
• a velocidade final do carrinho 2 é + 1 m/s.
Os sinais são muito importantes!! Muito cuidado!
ANTES DA COLISÃO
Os dois carrinhos se movimentam em sentidos opostos (sinais opostos), portanto,

DEPOIS DA COLISÃO
Os dois carrinhos se movimentam no mesmo sentido (sinais iguais), portanto,

Aplicando a Conservação da Quantidade de Movimento e substituindo os valores conhecidos.

3. Coeficiente de Restituição (e)

O Coeficiente de Restituição “e” mede a razão entre os módulos das velocidades relativas
de afastamento e de aproximação.

O conteúdo deste livro eletrônico é licenciado para DÉBORA LIZ RIBEIRO STELLE - , vedada, por quaisquer meios e a qualquer título,
a sua reprodução, cópia, divulgação ou distribuição, sujeitando-se aos infratores à responsabilização civil e criminal.
www.grancursosonline.com.br 29 de 140
FÍSICA
Conservação da Quantidade de Movimento e Colisões
Herico Avohai

Esse coeficiente é adimensional, dado em porcentagem, e mede a quantidade de energia


que foi transferida após a colisão.
O coeficiente de restituição tem valor mínimo igual a zero e valor máximo igual a 1 ou 100%.
Se:
- a colisão for perfeitamente elástica  e = 1;
Pois a velocidade relativa de afastamento é igual à velocidade relativa de aproximação e
há conservação de energia.
- a colisão for perfeitamente inelástica  e = 0.
Pois a velocidade relativa de afastamento é zero, ou seja, após a colisão os corpos perma-
necem grudados.
- a colisão for parcialmente elástica  0 < e < 1;
Pois não há conservação de Energia.
Pegando o exemplo do exemplo 4, vamos calcular o coeficiente de restituição.

Exemplo 4: A figura mostra o gráfico das velocidades de dois carrinhos que se movem sem
atrito sobre um mesmo par de trilhos horizontais e retilíneos. Em torno do instante 3 segundos,
os carrinhos colidem.

Determine o coeficiente de restituição.

O conteúdo deste livro eletrônico é licenciado para DÉBORA LIZ RIBEIRO STELLE - , vedada, por quaisquer meios e a qualquer título,
a sua reprodução, cópia, divulgação ou distribuição, sujeitando-se aos infratores à responsabilização civil e criminal.
www.grancursosonline.com.br 30 de 140
FÍSICA
Conservação da Quantidade de Movimento e Colisões
Herico Avohai

Já sabemos que:
• a velocidade inicial do carrinho 1 é – 2 m/s;
• a velocidade inicial do carrinho 2 é + 4 m/s;
• a velocidade final do carrinho 1 é + 3 m/s;
• a velocidade final do carrinho 2 é + 1 m/s.

Esquematizando o problema e considerando o referencial positivo para direita, temos:


ANTES DA COLISÃO

O módulo da velocidade relativa de aproximação de 1 em relação a 2 será:

DEPOIS DA COLISÃO

O conteúdo deste livro eletrônico é licenciado para DÉBORA LIZ RIBEIRO STELLE - , vedada, por quaisquer meios e a qualquer título,
a sua reprodução, cópia, divulgação ou distribuição, sujeitando-se aos infratores à responsabilização civil e criminal.
www.grancursosonline.com.br 31 de 140
FÍSICA
Conservação da Quantidade de Movimento e Colisões
Herico Avohai

O módulo da velocidade relativa de aproximação de 1 em relação a 2 será:

Utilizando a fórmula do coeficiente de restituição,

Significa que 33,33% da energia foi transferida após a interação entre os corpos.

Exemplo 5: (CESGRANRIO-RJ) Observa-se uma colisão elástica e unidimensional de uma partí-


cula de massa m e velocidade de módulo 5 m/s com outra partícula de massa m/4, inicialmente
em repouso. Quais são os valores dos módulos das velocidades das partículas após a colisão?

v’1 = 3 m/s e v’2 = 8 m/s


DADOS
ANTES
m1 = m
v1 = 5 m/s
m2 = m/4
v2 = 0
A questão diz que a colisão é elástica, então te pergunto, por que é importante esse dado?

O conteúdo deste livro eletrônico é licenciado para DÉBORA LIZ RIBEIRO STELLE - , vedada, por quaisquer meios e a qualquer título,
a sua reprodução, cópia, divulgação ou distribuição, sujeitando-se aos infratores à responsabilização civil e criminal.
www.grancursosonline.com.br 32 de 140
FÍSICA
Conservação da Quantidade de Movimento e Colisões
Herico Avohai

Muito bem, Bizurado! Exatamente isso, portanto vamos utilizar todas as características
desse tipo de colisão para encontrar as velocidades após o contato.

Adotando o referencial será positivo para direita.


Temos que o Coeficiente de restituição é dado por,

O conteúdo deste livro eletrônico é licenciado para DÉBORA LIZ RIBEIRO STELLE - , vedada, por quaisquer meios e a qualquer título,
a sua reprodução, cópia, divulgação ou distribuição, sujeitando-se aos infratores à responsabilização civil e criminal.
www.grancursosonline.com.br 33 de 140
FÍSICA
Conservação da Quantidade de Movimento e Colisões
Herico Avohai

Aplicando a conservação da Quantidade de Movimento, temos,

Cortando os “m’s” pois fazem parte de todos os termos,

Isolando v’1 na equação I e substituindo da equação II,

Equação II,

O conteúdo deste livro eletrônico é licenciado para DÉBORA LIZ RIBEIRO STELLE - , vedada, por quaisquer meios e a qualquer título,
a sua reprodução, cópia, divulgação ou distribuição, sujeitando-se aos infratores à responsabilização civil e criminal.
www.grancursosonline.com.br 34 de 140
FÍSICA
Conservação da Quantidade de Movimento e Colisões
Herico Avohai

Substituindo o valor de na equação I,

Ufa, o conteúdo acabou, agora é partir para exercitar a mente!!

O conteúdo deste livro eletrônico é licenciado para DÉBORA LIZ RIBEIRO STELLE - , vedada, por quaisquer meios e a qualquer título,
a sua reprodução, cópia, divulgação ou distribuição, sujeitando-se aos infratores à responsabilização civil e criminal.
www.grancursosonline.com.br 35 de 140
FÍSICA
Conservação da Quantidade de Movimento e Colisões
Herico Avohai

QUESTÕES DE CONCURSO
Questão 1 (EXÉRCITO/CADETE/ESPCEX/2011) Um canhão, inicialmente em repouso, de
massa 600 kg, dispara um projétil de massa 3 kg com velocidade horizontal de 800 m/s. Des-
prezando todos os atritos, podemos afirmar que a velocidade de recuo do canhão é de:
a) 2 m/s
b) 4 m/s
c) 6 m/s
d) 8 m/s
e) 12 m/s

Questão 2 (IDECAN/PROFESSOR/SEARH-RN/2016) Uma pessoa e uma carga de 50 kg


encontram-se em movimento retilíneo uniforme em um trenó cuja massa é de 125 kg. Consi-
dere que o trenó se desloca num plano horizontal com velocidade de 10 m/s e que num dado
instante a pessoa arremessa a carga para trás com velocidade de 9 m/s fazendo com que o
trenó passe a se deslocar com velocidade de 15 m/s. A massa dessa pessoa é de:
a) 60 kg.
b) 65 kg.
c) 70 kg.
d) 75 kg.

Questão 3 (FUNIVERSA/PERITO CRIMINAL/POLÍCIA CIENTÍFICA-GO/2015) Um policial


necessita de instruções e cursos para o aprimoramento de técnicas de tiro. Durante a ins-
trução, e (ou) curso, o policial adquire conhecimentos necessários para o correto manuseio
de armas no intuito de buscar a padronização de procedimentos operacionais bem como de
garantir maior segurança, tanto para o policial, quanto para os envolvidos na ação policial.
Durante a instrução, os policiais efetuam disparos de várias distâncias e enfrentam situações
que se assemelham à realidade encontrada no serviço diário visando ao melhor atendimento,
durante ocorrências, envolvendo confronto armado.

Internet: <www.blogdecastro.com>. Acesso em 2/1/2015 (com adaptações).

O conteúdo deste livro eletrônico é licenciado para DÉBORA LIZ RIBEIRO STELLE - , vedada, por quaisquer meios e a qualquer título,
a sua reprodução, cópia, divulgação ou distribuição, sujeitando-se aos infratores à responsabilização civil e criminal.
www.grancursosonline.com.br 36 de 140
FÍSICA
Conservação da Quantidade de Movimento e Colisões
Herico Avohai

Ao efetuar um disparo, o recuo da arma está relacionado com o princípio da conservação do


momento linear (quantidade de movimento). Nesse caso, o recuo ocorre para compensar a
quantidade de movimento adquirido pelo projétil. Normalmente, a quantidade de movimento
do retrocesso é absorvida pelo corpo do policial que dispara a arma. Com base no princípio da
conservação do momento linear, uma das armas utilizadas pela polícia brasileira é a Carabina
ponto 40. Supondo que essa arma, quando completamente carregada, tenha massa de 3,5 kg
e dispare um projétil de massa 10,0 g com velocidade inicial estimada em 450 m.s-1, assinale a
alternativa que apresenta o valor, aproximado, em metros por segundo, da velocidade de recuo
da arma.
a) 2/7 m.s-1
b) 2/9 m.s-1
c) 7/9 m.s-1
d) 9/7 m.s-1
e) 10/9 m.s-1

Questão 4 (EXÉRCITO/IME/2017) Um veículo de combate tem, como armamento principal,


um canhão automático eletromagnético, o qual está municiado com 50 projéteis. Esse veículo
se desloca em linha reta, inicialmente, em velocidade constante sobre um plano horizontal.
Como o veículo está sem freio e descontrolado, um engenheiro sugeriu executar disparos a fim
de reduzir a velocidade do veículo. Após realizar 10 disparos na mesma direção e no mesmo
sentido da velocidade inicial do veículo, este passou a se deslocar com metade da velocidade
inicial. Diante do exposto, a massa do veículo, em kg, é:
DADOS:
• velocidade inicial do veículo: 20 m/s;
• velocidade do projétil ao sair do canhão: 800 m/s; e
• massa do projétil: 2 kg.
Observação:
• não há atrito entre o plano horizontal e o veículo.

O conteúdo deste livro eletrônico é licenciado para DÉBORA LIZ RIBEIRO STELLE - , vedada, por quaisquer meios e a qualquer título,
a sua reprodução, cópia, divulgação ou distribuição, sujeitando-se aos infratores à responsabilização civil e criminal.
www.grancursosonline.com.br 37 de 140
FÍSICA
Conservação da Quantidade de Movimento e Colisões
Herico Avohai

a) 1.420
b) 1.480
c) 1.500
d) 1.580
e) 1.680

Questão 5 (FCC/PROFESSOR/SEDU-ES/2016) Um homem de massa 83 kg está parado


sobre um pequeno vagão de massa 1000 kg, em repouso nos trilhos horizontais. O homem dá
alguns passos e salta numa direção paralela aos trilhos com velocidade de 6,0 m/s. Imediata-
mente após o salto, a velocidade do vagão tem módulo, em m/s,
a) 0,50
b) 3,0
c) 1,5
d) 2,0
e) 1,0

Questão 6 (IF/PROFESSOR/IF-DF/2017) Um projétil de massa m e velocidade inicial Vo


atinge um bloco de massa M inicialmente parado. O projétil atravessa o bloco e sai com uma
velocidade igual a Vo/3 no mesmo sentido da velocidade inicial. Qual é a velocidade final do
bloco?
a) m.Vo / 3.M
b) 2.m.Vo / 3.M
c) 2.m.Vo / M
d) 3.m.Vo / M
e) 3.m.Vo / 2.M

Questão 7 (COSEAC/TÉCNICO DE LABORATÓRIO/UFF/2017) Uma prancha de massa M


está inicialmente em repouso sobre uma superfície horizontal. Na extremidade A dessa pran-
cha, encontra-se, também em repouso, um automóvel de massa m, assimilável a um ponto
material.

O conteúdo deste livro eletrônico é licenciado para DÉBORA LIZ RIBEIRO STELLE - , vedada, por quaisquer meios e a qualquer título,
a sua reprodução, cópia, divulgação ou distribuição, sujeitando-se aos infratores à responsabilização civil e criminal.
www.grancursosonline.com.br 38 de 140
FÍSICA
Conservação da Quantidade de Movimento e Colisões
Herico Avohai

A partir de certo instante, o automóvel passa a realizar um movimento em relação à superfície


horizontal, indo da extremidade A à extremidade B e, em marcha a ré, da extremidade B à ex-
tremidade A. Considere L o comprimento da prancha, µ o coeficiente de atrito estático entre os
pneus e a prancha e g a intensidade do campo gravitacional. Despreze o atrito entre a prancha
e a superfície em que se apoia. Nessas condições, o valor mínimo x do comprimento da pran-
cha, a fim de que o carro NÃO caia na superfície horizontal, é:
a) L
b) L/2
c) m.L/M+m
d) M.L/4
e) M.L/M+m

(CEBRASPE/PROFESSOR/SE-DF/2017)

Quando um foguete se movimenta no espaço vazio, seu momento é modificado porque parte
de sua massa é eliminada na forma de gases ejetados. Como esses gases adquirem algum
momento, o foguete recebe um momento compensador no sentido oposto, sendo, portanto,
acelerado como resultado da propulsão dos gases ejetados. As figuras apresentadas ilustram
o sistema de propulsão idealizado pelo cientista russo Konstantin Tsiolkovsky: um foguete de

O conteúdo deste livro eletrônico é licenciado para DÉBORA LIZ RIBEIRO STELLE - , vedada, por quaisquer meios e a qualquer título,
a sua reprodução, cópia, divulgação ou distribuição, sujeitando-se aos infratores à responsabilização civil e criminal.
www.grancursosonline.com.br 39 de 140
FÍSICA
Conservação da Quantidade de Movimento e Colisões
Herico Avohai

massa inicial m + Δm, que se desloca com velocidade v, sofre, em certo instante, um acrésci-
mo de velocidade Δv ao ejetar parte da sua massa (Δm) em alta velocidade (ve). A velocidade
inicial do foguete é muito menor que a velocidade da massa ejetada (v < ve).
Tendo como referência as informações precedentes, julgue os itens subsequentes, assumindo
que o momento linear do sistema se conserva e que as massas m e Δm não estão sujeitas a
forças externas ou de campo.

Questão 8 O acréscimo de velocidade adquirida pelo foguete devido à ejeção contínua de


sua massa depende das massas final e inicial do foguete.

Questão 9 A energia cinética do sistema é conservada — ou seja, permanece constante —


na direção do movimento mostrado nas figuras, devido à conservação do momento linear.

Questão 10 O momento linear total do sistema descrito é nulo no caso de o referencial estar
localizado no centro de massa do sistema.

Questão 11 (CESGRANRIO/ENGENHEIRO/PETROBRAS/2011) Suponha que dois corpos


de igual massa, se deslocando na mesma linha reta e na mesma direção, colidam e permane-
çam unidos após a colisão.
A velocidade comum após a colisão será
a) nula, se as velocidades iniciais dos corpos forem diferentes.
b) igual à do corpo inicialmente mais veloz.
c) a média simples das velocidades iniciais dos dois corpos.
d) a diferença entre as velocidades iniciais dos dois corpos.
e) a soma das velocidades dos dois corpos.

Questão 12 (COPESE/TÉCNICO DE LABORATÓRIO/UFJF/2017) Um experimento didático


consiste em dois carrinhos que podem se movimentar sem atrito em um trilho de ar. O movi-
mento da extremidade esquerda de cada um é descrito pelas tabelas abaixo, obtidas no mes-
mo sistema de referência.

O conteúdo deste livro eletrônico é licenciado para DÉBORA LIZ RIBEIRO STELLE - , vedada, por quaisquer meios e a qualquer título,
a sua reprodução, cópia, divulgação ou distribuição, sujeitando-se aos infratores à responsabilização civil e criminal.
www.grancursosonline.com.br 40 de 140
FÍSICA
Conservação da Quantidade de Movimento e Colisões
Herico Avohai

Sabendo que a massa do carrinho 1 é de 1 kg, e a do carrinho 2 é de 2 kg, e que ambos têm 10
cm de comprimento, podemos considerar que ocorreu o seguinte fenômeno:
a) Colisão completamente inelástica entre os dois carrinhos.
b) Colisão completamente elástica entre os dois carrinhos.
c) Os carrinhos não colidiram.
d) Houve uma colisão parcialmente inelástica.
e) Os dois carrinhos descrevem movimentos uniformemente acelerados.

Questão 13 (CEPERJ/PROFESSOR/SEDUC-RJ/2011)

Duas pequenas esferas de mesmas dimensões que se deslocam sobre uma guia horizontal,
com atritos desprezíveis, com movimento uniforme em sentidos opostos, vão colidir direta e
frontalmente. Antes da colisão, a esfera A, de massa igual a 3kg, move-se para a direita com
uma velocidade de módulo igual a 2m/s, enquanto que a esfera B, de massa igual a 1kg, move-
-se para a esquerda com uma velocidade de módulo igual a 10m/s, como ilustra a figura acima.
Se a colisão não for perfeitamente elástica, haverá um decréscimo no valor da energia cinética
do sistema constituído pelas duas esferas.
No caso dessa colisão, esse decréscimo poderá ser, no máximo de:
a) 12J
b) 24J
c) 44J

O conteúdo deste livro eletrônico é licenciado para DÉBORA LIZ RIBEIRO STELLE - , vedada, por quaisquer meios e a qualquer título,
a sua reprodução, cópia, divulgação ou distribuição, sujeitando-se aos infratores à responsabilização civil e criminal.
www.grancursosonline.com.br 41 de 140
FÍSICA
Conservação da Quantidade de Movimento e Colisões
Herico Avohai

d) 54J
e) 56J

Questão 14 (MARINHA/ESCOLA NAVAL/ASPIRANTE/2012) Um bloco A, de massa mA =1,0


kg, colide frontalmente com outro bloco, B, de massa mB =3,0 kg, que se encontrava inicialmen-
te em repouso. Para que os blocos sigam grudados com velocidade 2,0 m/s, a energia total
dissipada durante a colisão, em joules, deve ser
a) 24
b) 32
c) 36
d) 48
e) 64

Questão 15 (IDECAN/SOLDADO/CBM-DF/2017) Duas bolinhas A e B, com 0,5 kg cada, es-


tão se locomovendo na mesma direção e sentido em uma superfície sem atrito, como mostra
a figura a seguir:

Quando a bolinha A se encontra com a B, ambas movem-se juntas com velocidade igual a 5,25
m/s. Sabendo que VA = 2VB, então, a velocidade da bolinha A antes da colisão era:
a) 10,8 Km/h.
b) 18,72 Km/h.
c) 25,232 Km/h.
d) 27 Km/h.

O conteúdo deste livro eletrônico é licenciado para DÉBORA LIZ RIBEIRO STELLE - , vedada, por quaisquer meios e a qualquer título,
a sua reprodução, cópia, divulgação ou distribuição, sujeitando-se aos infratores à responsabilização civil e criminal.
www.grancursosonline.com.br 42 de 140
FÍSICA
Conservação da Quantidade de Movimento e Colisões
Herico Avohai

Questão 16 (MARINHA/ESCOLA NAVAL/ASPIRANTE/2011) A esfera de massa mo tem o


módulo da sua velocidade reduzida a zero na colisão frontal e inelástica (ou parcialmente elás-
tica) com a esfera de massa m = 2mo. Por sua vez, a esfera de massa m encontra-se inicialmen-
te em repouso na posição A, suspensa por um fio inextensível e de massa desprezível. Após a
colisão, percorre a trajetória circular ABCD de raio igual ao comprimento L do fio. Despreze o
atrito no pivô O e a resistência do ar. Para que a esfera de massa m percorra a trajetória circu-
lar, o valor mínimo do módulo da velocidade , antes da colisão, é Dado: g é a aceleração da
gravidade.

a) √g.L
b) √5g.L
c) √10g.L
d) 2√5g.L
e) 2√10g.L

Questão 17 (FCC/PROFESSOR/SED-ES/2016) Duas esferas A e B movem-se em direções


perpendiculares sobre uma mesa horizontal. As massas das esferas são mA = 1,5 kg e mB = 2,5
kg, respectivamente. Elas colidem anelasticamente e, no instante da colisão, suas velocidades
eram VA = 20 m/s e VB = 16 m/s. Imediatamente após a colisão, suas velocidades serão, res-
pectivamente,
a) 16 m/s e 20 m/s.
b) 20 m/s e 16 m/s.
c) 18 m/s e 18 m/s.

O conteúdo deste livro eletrônico é licenciado para DÉBORA LIZ RIBEIRO STELLE - , vedada, por quaisquer meios e a qualquer título,
a sua reprodução, cópia, divulgação ou distribuição, sujeitando-se aos infratores à responsabilização civil e criminal.
www.grancursosonline.com.br 43 de 140
FÍSICA
Conservação da Quantidade de Movimento e Colisões
Herico Avohai

d) 12,5 m/s e 12,5 m/s.


e) 18 m/s e 12,5 m/s.

Questão 18 (MARINHA/ENGENHEIRO/CEM/2015) Duas esferas A e B, com massas 0.2 Kg


e 0.3 Kg, respectivamente, movem-se numa reta orientada Ox com velocidades vA = 2 m/s e vB =
-3 m/s, respectivamente, e uma terceira esfera C, de massa 0.5 Kg, encontra-se em repouso na
origem. Num instante t > 0, as esferas A e B chocam-se com C. Após o choque, que é inelástico,
as três esferas movem-se juntas sobre a reta Ox. Nessas condições, após o choque as esferas
têm velocidade igual a
a) 1 m/s
b) 0.5 m/s
c) 0 m/s
d) -0.5 m/s
e) -1 m/s

Questão 19 (EXÉRCITO/ESPCEX/CADETE/2017) Uma granada de mão, inicialmente em re-


pouso, explode sobre uma mesa indestrutível, de superfície horizontal e sem atrito, e fragmen-
ta-se em três pedaços de massas m1, m2 e m3 que adquirem velocidades coplanares entre si e
paralelas ao plano da mesa.
Os valores das massas são m1 = m2 = m e m3 = m/2. Imediatamente após a explosão, as mas-
sas m1 e m2 adquirem as velocidades, respectivamente, cujos módulos são iguais a v, con-
forme o desenho abaixo. Desprezando todas as forças externas, o módulo da velocidade v3,
imediatamente após a explosão é

O conteúdo deste livro eletrônico é licenciado para DÉBORA LIZ RIBEIRO STELLE - , vedada, por quaisquer meios e a qualquer título,
a sua reprodução, cópia, divulgação ou distribuição, sujeitando-se aos infratores à responsabilização civil e criminal.
www.grancursosonline.com.br 44 de 140
FÍSICA
Conservação da Quantidade de Movimento e Colisões
Herico Avohai

a)

b)
c)

d)

e)

Questão 20 (AERONÁUTICA/OFICIAL/AFA/2017) Um corpo M de dimensões desprezíveis


e massa 10 kg movimentando-se em uma dimensão, inicialmente com velocidade, vai suces-
sivamente colidindo inelasticamente com N partículas m, todas de mesma massa 1 kg, e com
velocidades de módulo v = 20 m/s, que também se movimentam em uma dimensão de acordo
com a Figura 1, a seguir.

O gráfico que representa a velocidade final do conjunto vf após cada colisão em função do
número de partículas N é apresentado na Figura 2, a seguir.

Desconsiderando as forças de atrito e a resistência do ar sobre o corpo e as partículas, a colisão


de ordem N. na qual a velocidade do corpo resultante (corpo M + N. partículas m) se anula, é,
a) 25
b) 50
c) 100
d) 200

O conteúdo deste livro eletrônico é licenciado para DÉBORA LIZ RIBEIRO STELLE - , vedada, por quaisquer meios e a qualquer título,
a sua reprodução, cópia, divulgação ou distribuição, sujeitando-se aos infratores à responsabilização civil e criminal.
www.grancursosonline.com.br 45 de 140
FÍSICA
Conservação da Quantidade de Movimento e Colisões
Herico Avohai

Questão 21 (EXÉRCITO/ESPCEX/CADETE/2015) Dois caminhões de massa m1=2,0 ton e


m2=4,0 ton, com velocidades v1=30 m/s e v2=20 m/s, respectivamente, e trajetórias perpendicu-
lares entre si, colidem em um cruzamento no ponto G e passam a se movimentar unidos até o
ponto H, conforme a figura abaixo. Considerando o choque perfeitamente inelástico, o módulo
da velocidade dos veículos imediatamente após a colisão é:

a) 30 km/h
b) 40 km/h
c) 60 km/h
d) 70 km/h
e) 75 km/h

Questão 22 (FUNIVERSA/PERITO CRIMINAL/SPTC-GO/2010) O esquema representado na


figura a seguir é de um pêndulo balístico utilizado para periciar projéteis de armamentos. Uma
perícia realizada em dois projéteis de massas iguais a 10 g pode indicar a arma utilizada em
um crime em que o primeiro projétil, após ser disparado contra o pêndulo, deslocou este de
uma altura h = 20 cm. Considerando que o segundo projétil deslocou o pêndulo de uma altura
h = 45 cm e que a massa do bloco é M = 4 kg, assinale a alternativa correspondente às veloci-
dades dos projéteis v1 e v2. Assuma a aceleração da gravidade igual a 10 m/s2.

O conteúdo deste livro eletrônico é licenciado para DÉBORA LIZ RIBEIRO STELLE - , vedada, por quaisquer meios e a qualquer título,
a sua reprodução, cópia, divulgação ou distribuição, sujeitando-se aos infratores à responsabilização civil e criminal.
www.grancursosonline.com.br 46 de 140
FÍSICA
Conservação da Quantidade de Movimento e Colisões
Herico Avohai

a) v1 = 1.203 m/s e v2 = 802 m/s.


b) v1 = 1.203 m/h e v2 = 802 m/h.
c) v1 = 802 cm/s e v2 = 1.203 cm/s.
d) v1 = 802 m/s e v2 = 802 m/s.
e) v1 = 802 m/s e v2 = 1.203 m/s

Questão 23 (MARINHA/OFICIAL/2015) Um projétil de massa m = 20g é atirado horizontal-


mente com velocidade v0 contra um pêndulo vertical cuja massa pendular é M = 2Kg, de fácil
penetração. O projétil aloja-se no pêndulo e, devido ao choque, o conjunto sobe até a altura h =
20cm. Determine a velocidade inicial do projétil e assinale a opção correta.
Dado: g= 10m/s2
a) 200m/s
b) 202m/s
c) 204m/s
d) 206m/s

Questão 24 (EXÉRCITO/ESPCEX/CADETE/2018) Dois fios inextensíveis, paralelos, idênti-


cos e de massas desprezíveis suspendem um bloco regular de massa 10 kg formando um
pêndulo vertical balístico, inicialmente em repouso. Um projetil de massa igual a 100 g, com
velocidade horizontal, penetra e se aloja no bloco e, devido ao choque, o conjunto se eleva a
uma altura de 80 cm, conforme figura abaixo. Considere que os fios permaneçam sempre pa-
ralelos. A velocidade do projetil imediatamente antes de entrar no bloco é
Dados: despreze a resistência do ar e considere a aceleração da gravidade igual a 10 m/s2.

O conteúdo deste livro eletrônico é licenciado para DÉBORA LIZ RIBEIRO STELLE - , vedada, por quaisquer meios e a qualquer título,
a sua reprodução, cópia, divulgação ou distribuição, sujeitando-se aos infratores à responsabilização civil e criminal.
www.grancursosonline.com.br 47 de 140
FÍSICA
Conservação da Quantidade de Movimento e Colisões
Herico Avohai

a) 224 m/s.
b) 320 m/s.
c) 370 m/s.
d) 380 m/s.
e) 404 m/s.

Questão 25 (MARINHA/ESCOLA NAVAL/ASPIRANTE/2015) Analise a figura abaixo.

A figura acima mostra o gráfico das energias cinéticas de dois carrinhos, A e B respectivamen-
te, que deslizam sem atrito ao longo de um trilho horizontal retilíneo. No instante t=3s ocorre
uma colisão entre os carrinhos. Sendo assim, assinale a opção que pode representar um gráfi-
co para as velocidades dos carrinhos antes e depois da colisão.

O conteúdo deste livro eletrônico é licenciado para DÉBORA LIZ RIBEIRO STELLE - , vedada, por quaisquer meios e a qualquer título,
a sua reprodução, cópia, divulgação ou distribuição, sujeitando-se aos infratores à responsabilização civil e criminal.
www.grancursosonline.com.br 48 de 140
FÍSICA
Conservação da Quantidade de Movimento e Colisões
Herico Avohai

a)

b)

c)

d)

O conteúdo deste livro eletrônico é licenciado para DÉBORA LIZ RIBEIRO STELLE - , vedada, por quaisquer meios e a qualquer título,
a sua reprodução, cópia, divulgação ou distribuição, sujeitando-se aos infratores à responsabilização civil e criminal.
www.grancursosonline.com.br 49 de 140
FÍSICA
Conservação da Quantidade de Movimento e Colisões
Herico Avohai

e)

Questão 26 (MARINHA/ESCOLA NAVAL/ASPIRANTE/2011) Uma pista é composta por um


trecho retilíneo longo horizontal seguido do trecho circular vertical de raio R (conforme a figura
abaixo). O carrinho (1) (partícula), de massa m1 = 1,0 kg e velocidade = 5,0.î (m/ s), colide
com o carrinho (2) (partícula), de massa m2= 2,0kg, em repouso no trecho retilíneo. Despreze
os atritos. O coeficiente de restituição do choque vale 0,80. Após a colisão, o carrinho (2) sobe
o trecho circular vertical e, num certo instante, passa pela primeira vez na posição A, de altura
ha = R, com velocidade tal que o módulo da força normal da pista sobre o carrinho é igual ao
módulo do seu peso. Nesse instante, o módulo da velocidade (em m/ s) do carrinho (2) em
relação ao carrinho (1) é

a) 1,0
b) 1,2
c) 2,5
d) 2,0
e) 3,0

O conteúdo deste livro eletrônico é licenciado para DÉBORA LIZ RIBEIRO STELLE - , vedada, por quaisquer meios e a qualquer título,
a sua reprodução, cópia, divulgação ou distribuição, sujeitando-se aos infratores à responsabilização civil e criminal.
www.grancursosonline.com.br 50 de 140
FÍSICA
Conservação da Quantidade de Movimento e Colisões
Herico Avohai

Questão 27 (NUCEPE/PERITO CRIMINAL/PC-PI/2012) Um atirador próximo dispara com


uma espingarda de ar comprimido um pequeno chumbinho de borracha com massa igual a
0,002kg. O projétil desloca-se com velocidade aproximadamente horizontal igual a 50m/s e
atinge uma pequena partícula esférica de massa 0,03kg, inicialmente parada. A partícula esfé-
rica está posicionada na extremidade de uma corda de comprimento igual a L = 2,0 m, confor-
me pode ser visto na figura abaixo. Assumindo que a colisão entre o projétil e a partícula seja
perfeitamente elástica, determine qual deve ser a altura aproximada h adquirida pela partícula
após a colisão.

a) 2,00 metros
b) 1,95 metro
c) 1,65 metro
d) 1,50 metro
e) 1,25 metro

Questão 28 (AERONÁUTICA/ASPIRANTE/AFA/2011) De acordo com a figura abaixo, a par-


tícula A, ao ser abandonada de uma altura H, desce a rampa sem atritos ou resistência do ar
até sofrer uma colisão, perfeitamente elástica, com a partícula B que possui o dobro da massa
de A e que se encontra inicialmente em repouso. Após essa colisão, B entra em movimento e
A retorna, subindo a rampa e atingindo uma altura igual a

O conteúdo deste livro eletrônico é licenciado para DÉBORA LIZ RIBEIRO STELLE - , vedada, por quaisquer meios e a qualquer título,
a sua reprodução, cópia, divulgação ou distribuição, sujeitando-se aos infratores à responsabilização civil e criminal.
www.grancursosonline.com.br 51 de 140
FÍSICA
Conservação da Quantidade de Movimento e Colisões
Herico Avohai

a) H
b) H/2
c) H/3
d) H/9

Questão 29 (MARINHA/OFICIAL/EFOMM/2016) Dois móveis P e T com massas de 15,0 kg


e 13,0 kg, respectivamente, movem-se em sentidos opostos com velocidades VP = 5,0 m/s e
VT = 3,0 m/s, até sofrerem uma colisão unidimensional, parcialmente elástica de coeficiente
de restituição e = 3/4. Determine a intensidade de suas velocidades após o choque.
a) VT = 5 m/s e VP = 3,0 m/s
b) VT = 4,5 m/s e VP = 1,5 m/s
c) VT = 3,0 m/s e VP = 1,5 m/s
d) VT = 1,5 m/s e VP = 4,5 m/s
e) VT = 1,5 m/s e VP = 3,0 m/s

Questão 30 (IFB/PROFESSOR/IFB/2017) No alto de uma residência, apoia-se uma rampa


lisa na forma de um quadrante de circunferência de raio 0,45m. Do ponto A da rampa, abando-
na-se uma partícula de massa M que vai chocar-se elasticamente com outra partícula de mas-
sa 2M em repouso no ponto B, mais baixo da rampa. Considere g=10m/s2 e despreze todas
as formas de atrito. Determine os módulos das velocidades em m/s, com que as partículas
chegam ao solo.

O conteúdo deste livro eletrônico é licenciado para DÉBORA LIZ RIBEIRO STELLE - , vedada, por quaisquer meios e a qualquer título,
a sua reprodução, cópia, divulgação ou distribuição, sujeitando-se aos infratores à responsabilização civil e criminal.
www.grancursosonline.com.br 52 de 140
FÍSICA
Conservação da Quantidade de Movimento e Colisões
Herico Avohai

a) 1 e 2
b) 2 e 3
c) 3 e 4
d) 4 e 5
e) 5 e 6

Questão 31 (MARINHA/ESCOLA NAVAL/ASPIRANTE/2010) Dois pêndulos constituídos


por fios de massas desprezíveis e de comprimento L = 2,0 m estão pendurados em um teto em
dois pontos próximos de tal modo que as esferas A e B, de raios desprezíveis, estejam muito
próximas, sem se tocarem. As massas das esferas valem mA = 0,10 kg e mB = 0,15 kg. Aban-
dona-se a esfera A quando o fio forma um ângulo de 60º com a vertical, estando a esfera B do
outro pêndulo na posição de equilíbrio. Sabendo que, após a colisão frontal, a altura máxima
alcançada pelo centro de massa do sistema, em relação à posição de equilíbrio, é de 0,40 m, o
coeficiente de restituição da colisão é Dado: | g | = 10,0 m/ s 2

O conteúdo deste livro eletrônico é licenciado para DÉBORA LIZ RIBEIRO STELLE - , vedada, por quaisquer meios e a qualquer título,
a sua reprodução, cópia, divulgação ou distribuição, sujeitando-se aos infratores à responsabilização civil e criminal.
www.grancursosonline.com.br 53 de 140
FÍSICA
Conservação da Quantidade de Movimento e Colisões
Herico Avohai

a) zero
b) 0,25
c) 0,50
d) 0,75
e) 1,00

O conteúdo deste livro eletrônico é licenciado para DÉBORA LIZ RIBEIRO STELLE - , vedada, por quaisquer meios e a qualquer título,
a sua reprodução, cópia, divulgação ou distribuição, sujeitando-se aos infratores à responsabilização civil e criminal.
www.grancursosonline.com.br 54 de 140
FÍSICA
Conservação da Quantidade de Movimento e Colisões
Herico Avohai

GABARITO
1. b 28. d
2. b 29. b
3. d 30. a
4. b 31. e
5. a
6. b
7. e
8. C
9. E
10. C
11. c
12. b
13. d
14. a
15. c
16. d
17. d
18. d
19. e
20. b
21. c
22. e
23. b
24. e
25. a
26. d
27. b

O conteúdo deste livro eletrônico é licenciado para DÉBORA LIZ RIBEIRO STELLE - , vedada, por quaisquer meios e a qualquer título,
a sua reprodução, cópia, divulgação ou distribuição, sujeitando-se aos infratores à responsabilização civil e criminal.
www.grancursosonline.com.br 55 de 140
FÍSICA
Conservação da Quantidade de Movimento e Colisões
Herico Avohai

GABARITO COMENTADO
Questão 1 (EXÉRCITO/CADETE/ESPCEX/2011) Um canhão, inicialmente em repouso, de
massa 600 kg, dispara um projétil de massa 3 kg com velocidade horizontal de 800 m/s. Des-
prezando todos os atritos, podemos afirmar que a velocidade de recuo do canhão é de:
a) 2 m/s
b) 4 m/s
c) 6 m/s
d) 8 m/s
e) 12 m/s

Letra b.
Essa é aquela questão que eu deixei para você resolver no início da aula!! Espero que tenha
acertado!
DADOS
mc = 600 kg
vc = 0
mp = 3 kg
v’p = 800 m/s
v’c =?
Considerando o referencial positivo para direita e o sistema isolado, temos que a Quantidade
de Movimento se conserva.

ANTES
O conjunto canhão + projétil está em repouso, logo,

DEPOIS
O projétil é disparado para direita e o canhão tem velocidade de recuo para esquerda, logo,

O conteúdo deste livro eletrônico é licenciado para DÉBORA LIZ RIBEIRO STELLE - , vedada, por quaisquer meios e a qualquer título,
a sua reprodução, cópia, divulgação ou distribuição, sujeitando-se aos infratores à responsabilização civil e criminal.
www.grancursosonline.com.br 56 de 140
FÍSICA
Conservação da Quantidade de Movimento e Colisões
Herico Avohai

Para encontrar o módulo da Quantidade de Movimento depois, temos que realizar a soma ve-
torial,

Então, o módulo do vetor Quantidade de Movimento depois será:

Substituindo os valores na equação da conservação da Quantidade de Movimento,

Questão 2 (IDECAN/PROFESSOR/SEARH-RN/2016) Uma pessoa e uma carga de 50 kg


encontram-se em movimento retilíneo uniforme em um trenó cuja massa é de 125 kg. Consi-

O conteúdo deste livro eletrônico é licenciado para DÉBORA LIZ RIBEIRO STELLE - , vedada, por quaisquer meios e a qualquer título,
a sua reprodução, cópia, divulgação ou distribuição, sujeitando-se aos infratores à responsabilização civil e criminal.
www.grancursosonline.com.br 57 de 140
FÍSICA
Conservação da Quantidade de Movimento e Colisões
Herico Avohai

dere que o trenó se desloca num plano horizontal com velocidade de 10 m/s e que num dado
instante a pessoa arremessa a carga para trás com velocidade de 9 m/s fazendo com que o
trenó passe a se deslocar com velocidade de 15 m/s. A massa dessa pessoa é de:
a) 60 kg.
b) 65 kg.
c) 70 kg.
d) 75 kg.

Letra b.
DADOS
ANTES
mP =?
mC = 50 kg
mT = 125 kg
vP+C+T = 10 m/s (para direita)
DEPOIS
VC = 9 m/s (para esquerda)
vP+T = 15 m/s (para direita)
Considerando o sistema (pessoa+trenó+carga) isolado e o referencial positivo para direita,
temos que a Quantidade de Movimento se conserva.
ANTES

O conjunto se movimenta para direita, logo,

O conteúdo deste livro eletrônico é licenciado para DÉBORA LIZ RIBEIRO STELLE - , vedada, por quaisquer meios e a qualquer título,
a sua reprodução, cópia, divulgação ou distribuição, sujeitando-se aos infratores à responsabilização civil e criminal.
www.grancursosonline.com.br 58 de 140
FÍSICA
Conservação da Quantidade de Movimento e Colisões
Herico Avohai

DEPOIS
O conjunto (Trenó+Pessoa) se movimenta para direita e a carga se movimenta para esquerda.

Para encontrar o módulo da Quantidade de Movimento depois, temos que realizar a soma ve-
torial, como os dois vetores estão na horizontal e em sentidos opostos, temos:

Substituindo os valores na equação da conservação da Quantidade de Movimento,

O conteúdo deste livro eletrônico é licenciado para DÉBORA LIZ RIBEIRO STELLE - , vedada, por quaisquer meios e a qualquer título,
a sua reprodução, cópia, divulgação ou distribuição, sujeitando-se aos infratores à responsabilização civil e criminal.
www.grancursosonline.com.br 59 de 140
FÍSICA
Conservação da Quantidade de Movimento e Colisões
Herico Avohai

Questão 3 (FUNIVERSA/PERITO CRIMINAL/POLÍCIA CIENTÍFICA-GO/2015) Um policial


necessita de instruções e cursos para o aprimoramento de técnicas de tiro. Durante a ins-
trução, e (ou) curso, o policial adquire conhecimentos necessários para o correto manuseio
de armas no intuito de buscar a padronização de procedimentos operacionais bem como de
garantir maior segurança, tanto para o policial, quanto para os envolvidos na ação policial.
Durante a instrução, os policiais efetuam disparos de várias distâncias e enfrentam situações
que se assemelham à realidade encontrada no serviço diário visando ao melhor atendimento,
durante ocorrências, envolvendo confronto armado.

Internet: <www.blogdecastro.com>. Acesso em 2/1/2015 (com adaptações).

Ao efetuar um disparo, o recuo da arma está relacionado com o princípio da conservação do


momento linear (quantidade de movimento). Nesse caso, o recuo ocorre para compensar a
quantidade de movimento adquirido pelo projétil. Normalmente, a quantidade de movimento
do retrocesso é absorvida pelo corpo do policial que dispara a arma. Com base no princípio da
conservação do momento linear, uma das armas utilizadas pela polícia brasileira é a Carabina
ponto 40. Supondo que essa arma, quando completamente carregada, tenha massa de 3,5 kg
e dispare um projétil de massa 10,0 g com velocidade inicial estimada em 450 m.s-1, assinale a
alternativa que apresenta o valor, aproximado, em metros por segundo, da velocidade de recuo
da arma.
a) 2/7 m.s-1
b) 2/9 m.s-1
c) 7/9 m.s-1
d) 9/7 m.s-1
e) 10/9 m.s-1

Letra d.
Graduado(a), note que o examinador tentou te ludibriar com textão! Aff!

O conteúdo deste livro eletrônico é licenciado para DÉBORA LIZ RIBEIRO STELLE - , vedada, por quaisquer meios e a qualquer título,
a sua reprodução, cópia, divulgação ou distribuição, sujeitando-se aos infratores à responsabilização civil e criminal.
www.grancursosonline.com.br 60 de 140
FÍSICA
Conservação da Quantidade de Movimento e Colisões
Herico Avohai

A parte importante da questão é a partir de “Ao efetuar um disparo...”


DADOS
mcarabina = 3,5 kg
vc = 0
mp = 10 g = 0,01 kg
v’p = 450 m/s
v’c =?
Considerando o referencial positivo para direita e o sistema isolado, temos que a Quantidade
de Movimento se conserva.

ANTES
O conjunto carabina + projétil está em repouso, logo,

DEPOIS
O projétil é disparado para direita e a carabina tem recuo para esquerda, logo,

Para encontrar o módulo da Quantidade de Movimento depois, temos que realizar a soma ve-
torial,

O conteúdo deste livro eletrônico é licenciado para DÉBORA LIZ RIBEIRO STELLE - , vedada, por quaisquer meios e a qualquer título,
a sua reprodução, cópia, divulgação ou distribuição, sujeitando-se aos infratores à responsabilização civil e criminal.
www.grancursosonline.com.br 61 de 140
FÍSICA
Conservação da Quantidade de Movimento e Colisões
Herico Avohai

Então, o módulo do vetor Quantidade de Movimento depois será:

Substituindo os valores na equação da conservação da Quantidade de Movimento,

Questão 4 (EXÉRCITO/IME/2017) Um veículo de combate tem, como armamento principal,


um canhão automático eletromagnético, o qual está municiado com 50 projéteis. Esse veículo
se desloca em linha reta, inicialmente, em velocidade constante sobre um plano horizontal.
Como o veículo está sem freio e descontrolado, um engenheiro sugeriu executar disparos a fim
de reduzir a velocidade do veículo. Após realizar 10 disparos na mesma direção e no mesmo
sentido da velocidade inicial do veículo, este passou a se deslocar com metade da velocidade
inicial. Diante do exposto, a massa do veículo, em kg, é:

O conteúdo deste livro eletrônico é licenciado para DÉBORA LIZ RIBEIRO STELLE - , vedada, por quaisquer meios e a qualquer título,
a sua reprodução, cópia, divulgação ou distribuição, sujeitando-se aos infratores à responsabilização civil e criminal.
www.grancursosonline.com.br 62 de 140
FÍSICA
Conservação da Quantidade de Movimento e Colisões
Herico Avohai

DADOS:
velocidade inicial do veículo: 20 m/s;
velocidade do projétil ao sair do canhão: 800 m/s; e
massa do projétil: 2 kg.
Observação:
não há atrito entre o plano horizontal e o veículo.
a) 1.420
b) 1.480
c) 1.500
d) 1.580
e) 1.680

Letra b.
Não se assuste só porque a questão é do IME! Leia com atenção e pegue os dados.
DADOS
Mveículo =?
50 projéteis  m50 = 100 kg
Dez disparos  m10 = 20 kg
Vp = 800 m/s
V(V+50P) = 20 m/s (inicial)
V(V+40P) = 10 m/s (final)

Considerando o referencial positivo para direita e o sistema como isolado, pois só há interação
entre o veículo e os projéteis, temos que, temos que a Quantidade de Movimento se conserva.

O conteúdo deste livro eletrônico é licenciado para DÉBORA LIZ RIBEIRO STELLE - , vedada, por quaisquer meios e a qualquer título,
a sua reprodução, cópia, divulgação ou distribuição, sujeitando-se aos infratores à responsabilização civil e criminal.
www.grancursosonline.com.br 63 de 140
FÍSICA
Conservação da Quantidade de Movimento e Colisões
Herico Avohai

ANTES
O conjunto veículo + 50 projéteis está se movimentando para direita.

Logo a quantidade de movimento total antes será,

DEPOIS
10 projéteis são disparados para direita e pela ação e reação o veículo recebe a reação no sen-
tido contrário, diminuindo assim sua velocidade à metade.

Para encontrar o módulo da Quantidade de Movimento depois, temos que realizar a soma ve-
torial,

Então, o módulo do vetor Quantidade de Movimento depois será:

Substituindo os valores na equação da conservação da Quantidade de Movimento,

O conteúdo deste livro eletrônico é licenciado para DÉBORA LIZ RIBEIRO STELLE - , vedada, por quaisquer meios e a qualquer título,
a sua reprodução, cópia, divulgação ou distribuição, sujeitando-se aos infratores à responsabilização civil e criminal.
www.grancursosonline.com.br 64 de 140
FÍSICA
Conservação da Quantidade de Movimento e Colisões
Herico Avohai

Substituindo os valores conhecidos,

Multiplicando,

Gostou? Questão de nível difícil, mas tendo calma e paciência você consegue resolver.

Questão 5 (FCC/PROFESSOR/SEDU-ES/2016) Um homem de massa 83 kg está parado


sobre um pequeno vagão de massa 1000 kg, em repouso nos trilhos horizontais. O homem dá
alguns passos e salta numa direção paralela aos trilhos com velocidade de 6,0 m/s. Imediata-
mente após o salto, a velocidade do vagão tem módulo, em m/s,
a) 0,50
b) 3,0
c) 1,5
d) 2,0
e) 1,0

Letra a.
DADOS
mH = 83 kg
mV = 1000 kg
vH = 6,0 m/s
vV =?

O conteúdo deste livro eletrônico é licenciado para DÉBORA LIZ RIBEIRO STELLE - , vedada, por quaisquer meios e a qualquer título,
a sua reprodução, cópia, divulgação ou distribuição, sujeitando-se aos infratores à responsabilização civil e criminal.
www.grancursosonline.com.br 65 de 140
FÍSICA
Conservação da Quantidade de Movimento e Colisões
Herico Avohai

Questão sobre conservação da quantidade de movimento. Vamos analisar o problema antes e


depois do salto do homem.
Considerando o sistema (homem+vagão) isolado e o referencial positivo para direita, temos
que a Quantidade de Movimento se conserva.
ANTES

O conjunto (vagão+homem) está em repouso, logo a quantidade de movimento antes será,

DEPOIS
O homem salta para direita e o vagão se movimenta para esquerda.

Para encontrar o módulo da Quantidade de Movimento depois, temos que realizar a soma ve-
torial, como os dois vetores estão na horizontal e em sentidos opostos, temos:

O conteúdo deste livro eletrônico é licenciado para DÉBORA LIZ RIBEIRO STELLE - , vedada, por quaisquer meios e a qualquer título,
a sua reprodução, cópia, divulgação ou distribuição, sujeitando-se aos infratores à responsabilização civil e criminal.
www.grancursosonline.com.br 66 de 140
FÍSICA
Conservação da Quantidade de Movimento e Colisões
Herico Avohai

Substituindo os valores na equação da conservação da Quantidade de Movimento,

Questão 6 (IF/PROFESSOR/IF-DF/2017) Um projétil de massa m e velocidade inicial Vo


atinge um bloco de massa M inicialmente parado. O projétil atravessa o bloco e sai com uma
velocidade igual a Vo/3 no mesmo sentido da velocidade inicial. Qual é a velocidade final do
bloco?
a) m.Vo / 3.M
b) 2.m.Vo / 3.M
c) 2.m.Vo / M
d) 3.m.Vo / M
e) 3.m.Vo / 2.M

Letra b.
DADOS
mp = m
vp = V0
mb = M

O conteúdo deste livro eletrônico é licenciado para DÉBORA LIZ RIBEIRO STELLE - , vedada, por quaisquer meios e a qualquer título,
a sua reprodução, cópia, divulgação ou distribuição, sujeitando-se aos infratores à responsabilização civil e criminal.
www.grancursosonline.com.br 67 de 140
FÍSICA
Conservação da Quantidade de Movimento e Colisões
Herico Avohai

vb = 0
v’p = V0/3
v’B =?
Considerando o referencial positivo para direita e o sistema isolado, temos que a Quantidade
de Movimento se conserva.

ANTES

Somente o projétil se movimenta, logo,

E o seu módulo será,

DEPOIS
O projétil e o bloco se movimentam para direita, logo,

Para encontrar o módulo da Quantidade de Movimento depois, temos que realizar a soma ve-
torial, como os dois vetores estão na horizontal e para direita, basta somarmos,

O conteúdo deste livro eletrônico é licenciado para DÉBORA LIZ RIBEIRO STELLE - , vedada, por quaisquer meios e a qualquer título,
a sua reprodução, cópia, divulgação ou distribuição, sujeitando-se aos infratores à responsabilização civil e criminal.
www.grancursosonline.com.br 68 de 140
FÍSICA
Conservação da Quantidade de Movimento e Colisões
Herico Avohai

Substituindo os valores na equação da conservação da Quantidade de Movimento,

Questão 7 (COSEAC/TÉCNICO DE LABORATÓRIO/UFF/2017) Uma prancha de massa M


está inicialmente em repouso sobre uma superfície horizontal. Na extremidade A dessa pran-
cha, encontra-se, também em repouso, um automóvel de massa m, assimilável a um ponto
material.

A partir de certo instante, o automóvel passa a realizar um movimento em relação à superfície


horizontal, indo da extremidade A à extremidade B e, em marcha a ré, da extremidade B à ex-
tremidade A. Considere L o comprimento da prancha, µ o coeficiente de atrito estático entre os
pneus e a prancha e g a intensidade do campo gravitacional. Despreze o atrito entre a prancha
e a superfície em que se apoia. Nessas condições, o valor mínimo x do comprimento da pran-
cha, a fim de que o carro NÃO caia na superfície horizontal, é:
a) L
b) L/2
c) m.L/M+m

O conteúdo deste livro eletrônico é licenciado para DÉBORA LIZ RIBEIRO STELLE - , vedada, por quaisquer meios e a qualquer título,
a sua reprodução, cópia, divulgação ou distribuição, sujeitando-se aos infratores à responsabilização civil e criminal.
www.grancursosonline.com.br 69 de 140
FÍSICA
Conservação da Quantidade de Movimento e Colisões
Herico Avohai

d) M.L/4
e) M.L/M+m

Letra e.
DADOS
Comprimento da prancha L
mP = M
vp = 0
mA = m
vb = 0
Considerando o sistema (carro+plataforma) isolado e a origem do referencial no ponto A e
positivo para direita, temos que a Quantidade de Movimento se conserva.

ANTES
Tudo em repouso, logo,

DEPOIS
Quando o carro se movimenta para direita e percorre a distância x (comprimento da prancha),
a prancha se movimenta para esquerda percorrendo uma distância L – x, tudo em relação à
origem.

O conteúdo deste livro eletrônico é licenciado para DÉBORA LIZ RIBEIRO STELLE - , vedada, por quaisquer meios e a qualquer título,
a sua reprodução, cópia, divulgação ou distribuição, sujeitando-se aos infratores à responsabilização civil e criminal.
www.grancursosonline.com.br 70 de 140
FÍSICA
Conservação da Quantidade de Movimento e Colisões
Herico Avohai

Para encontrar o módulo da Quantidade de Movimento depois, temos que realizar a soma ve-
torial, como os dois vetores estão na horizontal e em sentidos opostos, temos:

Substituindo os valores na equação da conservação da Quantidade de Movimento,

Sabemos que,

O conteúdo deste livro eletrônico é licenciado para DÉBORA LIZ RIBEIRO STELLE - , vedada, por quaisquer meios e a qualquer título,
a sua reprodução, cópia, divulgação ou distribuição, sujeitando-se aos infratores à responsabilização civil e criminal.
www.grancursosonline.com.br 71 de 140
FÍSICA
Conservação da Quantidade de Movimento e Colisões
Herico Avohai

Substituindo na equação anterior, temos,

Como o intervalo de tempo é o mesmo para ambos,

O automóvel percorreu x e a prancha percorreu L – x, logo,

Colocando o x em evidência,

As questões sobre conservação da quantidade de movimento seguem esse padrão!! Refaça as


três primeiras questões antes de continuar.

(CEBRASPE/PROFESSOR/SE-DF/2017)

O conteúdo deste livro eletrônico é licenciado para DÉBORA LIZ RIBEIRO STELLE - , vedada, por quaisquer meios e a qualquer título,
a sua reprodução, cópia, divulgação ou distribuição, sujeitando-se aos infratores à responsabilização civil e criminal.
www.grancursosonline.com.br 72 de 140
FÍSICA
Conservação da Quantidade de Movimento e Colisões
Herico Avohai

Quando um foguete se movimenta no espaço vazio, seu momento é modificado porque parte
de sua massa é eliminada na forma de gases ejetados. Como esses gases adquirem algum
momento, o foguete recebe um momento compensador no sentido oposto, sendo, portanto,
acelerado como resultado da propulsão dos gases ejetados. As figuras apresentadas ilustram
o sistema de propulsão idealizado pelo cientista russo Konstantin Tsiolkovsky: um foguete de
massa inicial m + Δm, que se desloca com velocidade v, sofre, em certo instante, um acrésci-
mo de velocidade Δv ao ejetar parte da sua massa (Δm) em alta velocidade (ve). A velocidade
inicial do foguete é muito menor que a velocidade da massa ejetada (v < ve).
Tendo como referência as informações precedentes, julgue os itens subsequentes, assumindo
que o momento linear do sistema se conserva e que as massas m e Δm não estão sujeitas a
forças externas ou de campo.

Questão 8 O acréscimo de velocidade adquirida pelo foguete devido à ejeção contínua de


sua massa depende das massas final e inicial do foguete.

Questão 9 A energia cinética do sistema é conservada — ou seja, permanece constante —


na direção do movimento mostrado nas figuras, devido à conservação do momento linear.

Questão 10 O momento linear total do sistema descrito é nulo no caso de o referencial estar
localizado no centro de massa do sistema.

Certo, Errado, Certo.


DADOS
8. O acréscimo de velocidade adquirida pelo foguete devido à ejeção contínua de sua massa
depende das massas final e inicial do foguete.
Item certo, pois num sistema isolada o Momento Linear (Quantidade de Movimento) se con-
serva.

O conteúdo deste livro eletrônico é licenciado para DÉBORA LIZ RIBEIRO STELLE - , vedada, por quaisquer meios e a qualquer título,
a sua reprodução, cópia, divulgação ou distribuição, sujeitando-se aos infratores à responsabilização civil e criminal.
www.grancursosonline.com.br 73 de 140
FÍSICA
Conservação da Quantidade de Movimento e Colisões
Herico Avohai

Ou seja, depende das massas final e inicial do foguete.

9. A energia cinética do sistema é conservada — ou seja, permanece constante — na direção do


movimento mostrado nas figuras, devido à conservação do momento linear.
Item errado, como o enunciado não diz que a colisão é perfeitamente elástica, vimos, portanto,
que a Energia Cinética do sistema NÃO se conserva.

10. O momento linear total do sistema descrito é nulo no caso de o referencial estar localizado
no centro de massa do sistema.
Item certo. O enunciado diz que “assumindo que o momento linear do sistema se conserva e
que as massas m e Δm não estão sujeitas a forças externas ou de campo”, para um sistema
conservativo, o impulso é nulo, logo o somatório do momento linear (Quantidade de Movimen-
to Total) é nula também.

Questão 11 (CESGRANRIO/ENGENHEIRO/PETROBRAS/2011) Suponha que dois corpos


de igual massa, se deslocando na mesma linha reta e na mesma direção, colidam e permane-
çam unidos após a colisão.
A velocidade comum após a colisão será
a) nula, se as velocidades iniciais dos corpos forem diferentes.
b) igual à do corpo inicialmente mais veloz.
c) a média simples das velocidades iniciais dos dois corpos.
d) a diferença entre as velocidades iniciais dos dois corpos.
e) a soma das velocidades dos dois corpos.

Letra c.
DADOS
m1 = m2 = m
v1 ≠ 0

O conteúdo deste livro eletrônico é licenciado para DÉBORA LIZ RIBEIRO STELLE - , vedada, por quaisquer meios e a qualquer título,
a sua reprodução, cópia, divulgação ou distribuição, sujeitando-se aos infratores à responsabilização civil e criminal.
www.grancursosonline.com.br 74 de 140
FÍSICA
Conservação da Quantidade de Movimento e Colisões
Herico Avohai

v2 ≠ 0
v’ =? (Velocidade após a colisão)
Esquematizando o problema,
ANTES
Os dois corpos se movimentam na mesma direção e sentido. Coloquei sentido em negrito
porque é importante que os corpos estejam no mesmo sentido, ou então, o gabarito não seria
a letra c.
Se os corpos permanecem unidos após a colisão, então temos um tipo de colisão perfeitamen-
te inelástica.

Como os vetores estão na mesma direção e no mesmo sentido, a Quantidade de Movimento


antes terá módulo igual a,

DEPOIS DA COLISÃO
Os dois corpos ficam grudados e se movimentam com a mesma velocidade,

E o seu módulo será,

Sabemos que a Quantidade de Movimento se conserva, logo,

O conteúdo deste livro eletrônico é licenciado para DÉBORA LIZ RIBEIRO STELLE - , vedada, por quaisquer meios e a qualquer título,
a sua reprodução, cópia, divulgação ou distribuição, sujeitando-se aos infratores à responsabilização civil e criminal.
www.grancursosonline.com.br 75 de 140
FÍSICA
Conservação da Quantidade de Movimento e Colisões
Herico Avohai

Substituindo os valores conhecidos,

Colocando o “m” em evidência,

Questão 12 (COPESE/TÉCNICO DE LABORATÓRIO/UFJF/2017) Um experimento didático


consiste em dois carrinhos que podem se movimentar sem atrito em um trilho de ar. O movi-
mento da extremidade esquerda de cada um é descrito pelas tabelas abaixo, obtidas no mes-
mo sistema de referência.

Sabendo que a massa do carrinho 1 é de 1 kg, e a do carrinho 2 é de 2 kg, e que ambos têm 10
cm de comprimento, podemos considerar que ocorreu o seguinte fenômeno:
a) Colisão completamente inelástica entre os dois carrinhos.
b) Colisão completamente elástica entre os dois carrinhos.
c) Os carrinhos não colidiram.

O conteúdo deste livro eletrônico é licenciado para DÉBORA LIZ RIBEIRO STELLE - , vedada, por quaisquer meios e a qualquer título,
a sua reprodução, cópia, divulgação ou distribuição, sujeitando-se aos infratores à responsabilização civil e criminal.
www.grancursosonline.com.br 76 de 140
FÍSICA
Conservação da Quantidade de Movimento e Colisões
Herico Avohai

d) Houve uma colisão parcialmente inelástica.


e) Os dois carrinhos descrevem movimentos uniformemente acelerados.

Letra b.
DADOS
m1 = 1 kg
m2 = 2 kg
Observe que pela tabela o carrinho 1 se movimenta no sentido positivo do referencial, o car-
rinho 2 está em repouso e no instante t = 1 s, ocorre a colisão, logo em seguida, o carrinho 1
passa a se movimentar para o lado negativo do referencial e o carrinho 2 para o lado positivo.
Portanto, já sabemos que após a colisão, os corpos não permanecem grudados, que os ocorre
a colisão entre os carrinhos e os movimentos são retilíneos e uniformes (percorrem distâncias
iguais em intervalos de tempo iguais). Ficamos entre as letras b e d.
Vamos verificar se houve conservação da Energia cinética, caso positivo, a colisão é comple-
tamente elástica, caso negativo, a colisão é parcialmente inelástica.
ANTES
v1 ≠ 0
v2 = 0
Calculando a velocidade do carrinho 1 antes da colisão, entre os instantes t = 0 e t = 1,0 s,

Calculando e Energia Cinética antes da colisão,

O conteúdo deste livro eletrônico é licenciado para DÉBORA LIZ RIBEIRO STELLE - , vedada, por quaisquer meios e a qualquer título,
a sua reprodução, cópia, divulgação ou distribuição, sujeitando-se aos infratores à responsabilização civil e criminal.
www.grancursosonline.com.br 77 de 140
FÍSICA
Conservação da Quantidade de Movimento e Colisões
Herico Avohai

DEPOIS
v’1 ≠ 0
v’2 ≠ 0
Calculando as velocidades dos carrinhos 1 e 2 depois da colisão, entre os instantes t = 1,0 s e
t = 2,5 s,
Carrinho 1

O negativo significa que o carrinho 1 está se movimentando par ao lado negativo da trajetória.
Carrinho 2

A Energia Cinética Total depois da colisão será,

O conteúdo deste livro eletrônico é licenciado para DÉBORA LIZ RIBEIRO STELLE - , vedada, por quaisquer meios e a qualquer título,
a sua reprodução, cópia, divulgação ou distribuição, sujeitando-se aos infratores à responsabilização civil e criminal.
www.grancursosonline.com.br 78 de 140
FÍSICA
Conservação da Quantidade de Movimento e Colisões
Herico Avohai

A Energia cinética antes é igual à Energia cinética depois, logo, a colisão é completamente
elástica.

Questão 13 (CEPERJ/PROFESSOR/SEDUC-RJ/2011)

Duas pequenas esferas de mesmas dimensões que se deslocam sobre uma guia horizontal,
com atritos desprezíveis, com movimento uniforme em sentidos opostos, vão colidir direta e
frontalmente. Antes da colisão, a esfera A, de massa igual a 3kg, move-se para a direita com
uma velocidade de módulo igual a 2m/s, enquanto que a esfera B, de massa igual a 1kg, move-
-se para a esquerda com uma velocidade de módulo igual a 10m/s, como ilustra a figura acima.
Se a colisão não for perfeitamente elástica, haverá um decréscimo no valor da energia cinética
do sistema constituído pelas duas esferas.
No caso dessa colisão, esse decréscimo poderá ser, no máximo de:
a) 12J
b) 24J
c) 44J
d) 54J
e) 56J

Letra d.
DADOS
mA = 3 kg
mB = 1 kg
vA = 2 m/s
vB = 10 m/s
v’ =? (Velocidade após a colisão)

O conteúdo deste livro eletrônico é licenciado para DÉBORA LIZ RIBEIRO STELLE - , vedada, por quaisquer meios e a qualquer título,
a sua reprodução, cópia, divulgação ou distribuição, sujeitando-se aos infratores à responsabilização civil e criminal.
www.grancursosonline.com.br 79 de 140
FÍSICA
Conservação da Quantidade de Movimento e Colisões
Herico Avohai

O decréscimo de Energia será a diferença entre as Energias Cinéticas antes e depois.

Para o maior decréscimo, temos que o tipo de colisão tem que ser perfeitamente inelástica, ou

seja, quando os corpos após a colisão ficam grudados.

Portanto,

ANTES DA COLISÃO

Os dois corpos se movimentam na mesma direção e sentidos contrários.

A Quantidade de movimento antes será,

Como os vetores estão na mesma direção e em sentidos opostos, a Quantidade de Movimento

antes terá módulo igual a,

Calculando a Energia Cinética antes,

Substituindo os valores conhecidos,

DEPOIS DA COLISÃO
Os dois corpos ficam grudados e se movimentam com a mesma velocidade, (SITUAÇÃO COM
A MAIOR PERDA DE ENERGIA CINÉTICA)

O conteúdo deste livro eletrônico é licenciado para DÉBORA LIZ RIBEIRO STELLE - , vedada, por quaisquer meios e a qualquer título,
a sua reprodução, cópia, divulgação ou distribuição, sujeitando-se aos infratores à responsabilização civil e criminal.
www.grancursosonline.com.br 80 de 140
FÍSICA
Conservação da Quantidade de Movimento e Colisões
Herico Avohai

E o seu módulo será,

Sabemos que a Quantidade de Movimento se conserva, logo,

Substituindo os valores conhecidos,

A Energia Cinética depois será,

Substituindo os valores conhecidos,

O conteúdo deste livro eletrônico é licenciado para DÉBORA LIZ RIBEIRO STELLE - , vedada, por quaisquer meios e a qualquer título,
a sua reprodução, cópia, divulgação ou distribuição, sujeitando-se aos infratores à responsabilização civil e criminal.
www.grancursosonline.com.br 81 de 140
FÍSICA
Conservação da Quantidade de Movimento e Colisões
Herico Avohai

O decréscimo de Energia será,

Questão 14 (MARINHA/ESCOLA NAVAL/ASPIRANTE/2012) Um bloco A, de massa mA =1,0


kg, colide frontalmente com outro bloco, B, de massa mB =3,0 kg, que se encontrava inicialmen-
te em repouso. Para que os blocos sigam grudados com velocidade 2,0 m/s, a energia total
dissipada durante a colisão, em joules, deve ser
a) 24
b) 32
c) 36
d) 48
e) 64

Letra a.
DADOS
mA = 1,0 kg
mB = 3,0 kg
vconj = 2,0 m/s
vB0 = 0
A maior perda de energia será quando o choque for do tipo perfeitamente inelástico, ou seja,
após a colisão, os corpos permanecem grudados, conforme o enunciado.
Portanto,
ANTES DA COLISÃO
O bloco A se movimenta e o bloco B está em repouso,

O conteúdo deste livro eletrônico é licenciado para DÉBORA LIZ RIBEIRO STELLE - , vedada, por quaisquer meios e a qualquer título,
a sua reprodução, cópia, divulgação ou distribuição, sujeitando-se aos infratores à responsabilização civil e criminal.
www.grancursosonline.com.br 82 de 140
FÍSICA
Conservação da Quantidade de Movimento e Colisões
Herico Avohai

A Quantidade de movimento antes será,

Calculando a Energia Cinética antes,

Substituindo os valores conhecidos,

Temos que encontrar o valor da velocidade inicial de A.


DEPOIS DA COLISÃO
Os dois corpos ficam grudados e se movimentam com a mesma velocidade, (SITUAÇÃO COM
A MAIOR PERDA DE ENERGIA CINÉTICA)

O conteúdo deste livro eletrônico é licenciado para DÉBORA LIZ RIBEIRO STELLE - , vedada, por quaisquer meios e a qualquer título,
a sua reprodução, cópia, divulgação ou distribuição, sujeitando-se aos infratores à responsabilização civil e criminal.
www.grancursosonline.com.br 83 de 140
FÍSICA
Conservação da Quantidade de Movimento e Colisões
Herico Avohai

E o seu módulo será,

Sabemos que a Quantidade de Movimento se conserva, logo,

Substituindo os valores conhecidos,

A Energia Cinética depois será,

Substituindo os valores conhecidos,

E a energia antes,

O conteúdo deste livro eletrônico é licenciado para DÉBORA LIZ RIBEIRO STELLE - , vedada, por quaisquer meios e a qualquer título,
a sua reprodução, cópia, divulgação ou distribuição, sujeitando-se aos infratores à responsabilização civil e criminal.
www.grancursosonline.com.br 84 de 140
FÍSICA
Conservação da Quantidade de Movimento e Colisões
Herico Avohai

A perda de Energia será

Questão 15 (IDECAN/SOLDADO/CBM-DF/2017) Duas bolinhas A e B, com 0,5 kg cada, es-


tão se locomovendo na mesma direção e sentido em uma superfície sem atrito, como mostra
a figura a seguir:

Quando a bolinha A se encontra com a B, ambas movem-se juntas com velocidade igual a 5,25
m/s. Sabendo que VA = 2VB, então, a velocidade da bolinha A antes da colisão era:
a) 10,8 Km/h.
b) 18,72 Km/h.
c) 25,232 Km/h.
d) 27 Km/h.

Letra c.
DADOS
mA = 0,5 kg
mB = 0,5 kg
vCONJUNTO = 5,25 m/s
vA = 2vB
vA =?
ANTES DA COLISÃO
Os corpos A e B se movimentam na mesma direção e sentido,

O conteúdo deste livro eletrônico é licenciado para DÉBORA LIZ RIBEIRO STELLE - , vedada, por quaisquer meios e a qualquer título,
a sua reprodução, cópia, divulgação ou distribuição, sujeitando-se aos infratores à responsabilização civil e criminal.
www.grancursosonline.com.br 85 de 140
FÍSICA
Conservação da Quantidade de Movimento e Colisões
Herico Avohai

A Quantidade de movimento antes será,

DEPOIS DA COLISÃO
Os dois corpos ficam grudados e se movimentam com a mesma velocidade,

E o seu módulo será,

Sabemos que a Quantidade de Movimento se conserva, logo,

Substituindo os valores conhecidos,

O conteúdo deste livro eletrônico é licenciado para DÉBORA LIZ RIBEIRO STELLE - , vedada, por quaisquer meios e a qualquer título,
a sua reprodução, cópia, divulgação ou distribuição, sujeitando-se aos infratores à responsabilização civil e criminal.
www.grancursosonline.com.br 86 de 140
FÍSICA
Conservação da Quantidade de Movimento e Colisões
Herico Avohai

Sabemos que ,

Encontrando VA,

Questão 16 (MARINHA/ESCOLA NAVAL/ASPIRANTE/2011) A esfera de massa mo tem o


módulo da sua velocidade reduzida a zero na colisão frontal e inelástica (ou parcialmente elás-
tica) com a esfera de massa m = 2mo. Por sua vez, a esfera de massa m encontra-se inicialmen-
te em repouso na posição A, suspensa por um fio inextensível e de massa desprezível. Após
a colisão, percorre a trajetória circular ABCD de raio igual ao comprimento L do fio. Despreze
o atrito no pivô O e a resistência do ar. Para que a esfera de massa m percorra a trajetória cir-
cular, o valor mínimo do módulo da velocidade , antes da colisão, é Dado: g é a aceleração
da gravidade.

O conteúdo deste livro eletrônico é licenciado para DÉBORA LIZ RIBEIRO STELLE - , vedada, por quaisquer meios e a qualquer título,
a sua reprodução, cópia, divulgação ou distribuição, sujeitando-se aos infratores à responsabilização civil e criminal.
www.grancursosonline.com.br 87 de 140
FÍSICA
Conservação da Quantidade de Movimento e Colisões
Herico Avohai

a) √g.L
b) √5g.L
c) √10g.L
d) 2√5g.L
e) 2√10g.L

Letra d.
DADOS
m = 2mo
v0 =?
Para que a partícula percorra a trajetória circular, no ponto mais alto (ponto C) a força peso deve
ser igual a força centrípeta, ou seja, T = 0. Essa é a condição para o looping, tá lembrado(a)?

Agora, pensando na conservação de energia entre os pontos A e C, temos que no ponto A, a


partícula m só possui energia cinética, já no ponto C, ela possui energias cinética e potencial
gravitacional.

O conteúdo deste livro eletrônico é licenciado para DÉBORA LIZ RIBEIRO STELLE - , vedada, por quaisquer meios e a qualquer título,
a sua reprodução, cópia, divulgação ou distribuição, sujeitando-se aos infratores à responsabilização civil e criminal.
www.grancursosonline.com.br 88 de 140
FÍSICA
Conservação da Quantidade de Movimento e Colisões
Herico Avohai

Para calcular a velocidade inicial, vamos aplicar a conservação da quantidade de movimento.

Antes somente m0 se movimenta e depois somente m se movimenta.

Questão 17 (FCC/PROFESSOR/SED-ES/2016) Duas esferas A e B movem-se em direções


perpendiculares sobre uma mesa horizontal. As massas das esferas são mA = 1,5 kg e mB = 2,5
kg, respectivamente. Elas colidem anelasticamente e, no instante da colisão, suas velocidades
eram VA = 20 m/s e VB = 16 m/s. Imediatamente após a colisão, suas velocidades serão, res-
pectivamente,
a) 16 m/s e 20 m/s.
b) 20 m/s e 16 m/s.
c) 18 m/s e 18 m/s.
d) 12,5 m/s e 12,5 m/s.
e) 18 m/s e 12,5 m/s.

O conteúdo deste livro eletrônico é licenciado para DÉBORA LIZ RIBEIRO STELLE - , vedada, por quaisquer meios e a qualquer título,
a sua reprodução, cópia, divulgação ou distribuição, sujeitando-se aos infratores à responsabilização civil e criminal.
www.grancursosonline.com.br 89 de 140
FÍSICA
Conservação da Quantidade de Movimento e Colisões
Herico Avohai

Letra d.
DADOS
mA = 1,5 kg
mB = 2,5 kg
vA = 20 m/s
vB = 16 m/s
Por ser choque anelástico, já sabemos que após a colisão os corpos se movimentam grudados
e com a mesma velocidade. Nesse caso, já poderíamos excluir as alternativas “a”, “b” e “e”.
Gostei dessa questão, Comando!!! Vamos esquematizá-la?
ANTES DA COLISÃO
Os corpos A e B se movimentam perpendicularmente entre si,

A Quantidade de movimento antes será,

Note que os vetores formam 90º, portanto o módulo da soma vetorial será encontrado pelo
Teorema de Pitágoras.
Soma vetorial será dada por,

O conteúdo deste livro eletrônico é licenciado para DÉBORA LIZ RIBEIRO STELLE - , vedada, por quaisquer meios e a qualquer título,
a sua reprodução, cópia, divulgação ou distribuição, sujeitando-se aos infratores à responsabilização civil e criminal.
www.grancursosonline.com.br 90 de 140
FÍSICA
Conservação da Quantidade de Movimento e Colisões
Herico Avohai

Aplicando o Teorema de Pitágoras,

DEPOIS DA COLISÃO
Os dois corpos ficam grudados e se movimentam com a mesma velocidade,

E o seu módulo será,

Sabemos que a Quantidade de Movimento se conserva, logo,

Substituindo os valores conhecidos,

O conteúdo deste livro eletrônico é licenciado para DÉBORA LIZ RIBEIRO STELLE - , vedada, por quaisquer meios e a qualquer título,
a sua reprodução, cópia, divulgação ou distribuição, sujeitando-se aos infratores à responsabilização civil e criminal.
www.grancursosonline.com.br 91 de 140
FÍSICA
Conservação da Quantidade de Movimento e Colisões
Herico Avohai

Questão 18 (MARINHA/ENGENHEIRO/CEM/2015) Duas esferas A e B, com massas 0.2 Kg


e 0.3 Kg, respectivamente, movem-se numa reta orientada Ox com velocidades vA = 2 m/s e vB =
-3 m/s, respectivamente, e uma terceira esfera C, de massa 0.5 Kg, encontra-se em repouso na
origem. Num instante t > 0, as esferas A e B chocam-se com C. Após o choque, que é inelástico,
as três esferas movem-se juntas sobre a reta Ox. Nessas condições, após o choque as esferas
têm velocidade igual a
a) 1 m/s
b) 0.5 m/s
c) 0 m/s
d) -0.5 m/s
e) -1 m/s

Letra d.
DADOS
mA = 0,2 kg
mB = 0,3 kg
mC = 0,5 kg
vA = 2 m/s
vB = -3 m/s
vC = 0

O conteúdo deste livro eletrônico é licenciado para DÉBORA LIZ RIBEIRO STELLE - , vedada, por quaisquer meios e a qualquer título,
a sua reprodução, cópia, divulgação ou distribuição, sujeitando-se aos infratores à responsabilização civil e criminal.
www.grancursosonline.com.br 92 de 140
FÍSICA
Conservação da Quantidade de Movimento e Colisões
Herico Avohai

ANTES DA COLISÃO
Os corpos A e B se movimentam e C está em repouso,

A Quantidade de movimento antes será,

Respeitando os vetores, temos que o módulo da Quantidade de Movimento antes será,

DEPOIS DA COLISÃO
Os três corpos ficam grudados e se movimentam com a mesma velocidade,

E o seu módulo será,

Sabemos que a Quantidade de Movimento se conserva, logo,

O conteúdo deste livro eletrônico é licenciado para DÉBORA LIZ RIBEIRO STELLE - , vedada, por quaisquer meios e a qualquer título,
a sua reprodução, cópia, divulgação ou distribuição, sujeitando-se aos infratores à responsabilização civil e criminal.
www.grancursosonline.com.br 93 de 140
FÍSICA
Conservação da Quantidade de Movimento e Colisões
Herico Avohai

Substituindo os valores conhecidos,

Questão 19 (EXÉRCITO/ESPCEX/CADETE/2017) Uma granada de mão, inicialmente em re-


pouso, explode sobre uma mesa indestrutível, de superfície horizontal e sem atrito, e fragmen-
ta-se em três pedaços de massas m1, m2 e m3 que adquirem velocidades coplanares entre si e
paralelas ao plano da mesa.
Os valores das massas são m1 = m2 = m e m3 = m/2. Imediatamente após a explosão, as mas-
sas m1 e m2 adquirem as velocidades, respectivamente, cujos módulos são iguais a v, con-
forme o desenho abaixo. Desprezando todas as forças externas, o módulo da velocidade v3,
imediatamente após a explosão é

a)

b)
c)

d)

e)

O conteúdo deste livro eletrônico é licenciado para DÉBORA LIZ RIBEIRO STELLE - , vedada, por quaisquer meios e a qualquer título,
a sua reprodução, cópia, divulgação ou distribuição, sujeitando-se aos infratores à responsabilização civil e criminal.
www.grancursosonline.com.br 94 de 140
FÍSICA
Conservação da Quantidade de Movimento e Colisões
Herico Avohai

Letra e.
DADOS
m1 = m
m2 = m
m3 = m/2
v1 = v
v2 = v
vC =?
No sistema isolado a Quantidade de Movimento Total se conserva. Inicialmente não há mo-
vimento, portanto, a quantidade de movimento total é nula, pois não há forças externas atu-
ando.

Depois da explosão, as massas 1 e 2 movimentam-se perpendicularmente entre si e a massa


3 se movimenta na diagonal para o Sudoeste.

Como sabemos que a Quantidade de Movimento Total é nula, podemos concluir que o somató-
rio das Quantidades de Movimentos de 1 e 2 será igual à Quantidade de Movimento de 3.

Note que os vetores formam 90º, portanto o módulo da soma vetorial será encontrado pelo
Teorema de Pitágoras.

O conteúdo deste livro eletrônico é licenciado para DÉBORA LIZ RIBEIRO STELLE - , vedada, por quaisquer meios e a qualquer título,
a sua reprodução, cópia, divulgação ou distribuição, sujeitando-se aos infratores à responsabilização civil e criminal.
www.grancursosonline.com.br 95 de 140
FÍSICA
Conservação da Quantidade de Movimento e Colisões
Herico Avohai

Questão 20 (AERONÁUTICA/OFICIAL/AFA/2017) Um corpo M de dimensões desprezíveis


e massa 10 kg movimentando-se em uma dimensão, inicialmente com velocidade, vai suces-
sivamente colidindo inelasticamente com N partículas m, todas de mesma massa 1 kg, e com
velocidades de módulo v = 20 m/s, que também se movimentam em uma dimensão de acordo
com a Figura 1, a seguir.

O gráfico que representa a velocidade final do conjunto vf após cada colisão em função do
número de partículas N é apresentado na Figura 2, a seguir.

O conteúdo deste livro eletrônico é licenciado para DÉBORA LIZ RIBEIRO STELLE - , vedada, por quaisquer meios e a qualquer título,
a sua reprodução, cópia, divulgação ou distribuição, sujeitando-se aos infratores à responsabilização civil e criminal.
www.grancursosonline.com.br 96 de 140
FÍSICA
Conservação da Quantidade de Movimento e Colisões
Herico Avohai

Desconsiderando as forças de atrito e a resistência do ar sobre o corpo e as partículas, a coli-


são de ordem N. na qual a velocidade do corpo resultante (corpo M + N. partículas m) se anula,
é,
a) 25
b) 50
c) 100
d) 200

Letra b.
DADOS
M = 10 kg
m = 1 kg
vm = 20 m/s
v2 = v
vC =?
Minha(meu) cara(o) amiga(o)!! Que questão é essa!!??? Cavernosa! Último nível! TOP das ga-
láxias! Essa é do nível final de campeonato!! Mas não tenha medo, aplique os conhecimentos
que ela sairá!!
Primeiro temos que encontrar a velocidade inicial de M e em seguida calcular o valor de n.
Sabemos que num sistema isolado, a quantidade de movimento se conserva, logo,

O conteúdo deste livro eletrônico é licenciado para DÉBORA LIZ RIBEIRO STELLE - , vedada, por quaisquer meios e a qualquer título,
a sua reprodução, cópia, divulgação ou distribuição, sujeitando-se aos infratores à responsabilização civil e criminal.
www.grancursosonline.com.br 97 de 140
FÍSICA
Conservação da Quantidade de Movimento e Colisões
Herico Avohai

ANTES DA COLISÃO
O corpo maior se movimenta para direita e os corpos menores se movimentam para esquerda,
então,

DEPOIS DA COLISÃO
Os corpos se movimentam grudados, logo,

Aplicando na conservação da Quantidade de Movimento, temos,

Olhando o gráfico, temos que:


Se n = 10, vfinal = 40 m/s
Substituindo os valores conhecidos na equação acima,

O examinador pede a quantidade de massas “n” para que a velocidade final do conjunto seja
zero, logo,

O conteúdo deste livro eletrônico é licenciado para DÉBORA LIZ RIBEIRO STELLE - , vedada, por quaisquer meios e a qualquer título,
a sua reprodução, cópia, divulgação ou distribuição, sujeitando-se aos infratores à responsabilização civil e criminal.
www.grancursosonline.com.br 98 de 140
FÍSICA
Conservação da Quantidade de Movimento e Colisões
Herico Avohai

Questão 21 (EXÉRCITO/ESPCEX/CADETE/2015) Dois caminhões de massa m1=2,0 ton e


m2=4,0 ton, com velocidades v1=30 m/s e v2=20 m/s, respectivamente, e trajetórias perpendicu-
lares entre si, colidem em um cruzamento no ponto G e passam a se movimentar unidos até o
ponto H, conforme a figura abaixo. Considerando o choque perfeitamente inelástico, o módulo
da velocidade dos veículos imediatamente após a colisão é:

a) 30 km/h
b) 40 km/h
c) 60 km/h
d) 70 km/h
e) 75 km/h

Letra c.
Olha que blz! Já tô te vendo na academia!!
DADOS

O conteúdo deste livro eletrônico é licenciado para DÉBORA LIZ RIBEIRO STELLE - , vedada, por quaisquer meios e a qualquer título,
a sua reprodução, cópia, divulgação ou distribuição, sujeitando-se aos infratores à responsabilização civil e criminal.
www.grancursosonline.com.br 99 de 140
FÍSICA
Conservação da Quantidade de Movimento e Colisões
Herico Avohai

m1 = 2,0 ton = 2000 kg = 2x103 kg


m2 = 4,0 ton = 4000 kg = 4x103 kg
v1 = 30 m/s
v2 = 20 m/s
Por ser choque perfeitamente inelástico, já sabemos que após a colisão os caminhões se mo-
vimentam grudados e com a mesma velocidade.
Vamos esquematizá-la?
ANTES DA COLISÃO
Os caminhões 1 e 2 se movimentam perpendicularmente entre si,

A Quantidade de movimento antes será,

Note que os vetores formam 90º, portanto o módulo da soma vetorial será encontrado pelo
Teorema de Pitágoras.
Soma vetorial será dada por,

O conteúdo deste livro eletrônico é licenciado para DÉBORA LIZ RIBEIRO STELLE - , vedada, por quaisquer meios e a qualquer título,
a sua reprodução, cópia, divulgação ou distribuição, sujeitando-se aos infratores à responsabilização civil e criminal.
www.grancursosonline.com.br 100 de 140
FÍSICA
Conservação da Quantidade de Movimento e Colisões
Herico Avohai

Aplicando o Teorema de Pitágoras,

DEPOIS DA COLISÃO
Os dois corpos ficam grudados e se movimentam com a mesma velocidade,

E o seu módulo será,

Sabemos que a Quantidade de Movimento se conserva, logo,

O conteúdo deste livro eletrônico é licenciado para DÉBORA LIZ RIBEIRO STELLE - , vedada, por quaisquer meios e a qualquer título,
a sua reprodução, cópia, divulgação ou distribuição, sujeitando-se aos infratores à responsabilização civil e criminal.
www.grancursosonline.com.br 101 de 140
FÍSICA
Conservação da Quantidade de Movimento e Colisões
Herico Avohai

Substituindo os valores conhecidos,

Transformando para km/h,

Questão 22 (FUNIVERSA/PERITO CRIMINAL/SPTC-GO/2010) O esquema representado na


figura a seguir é de um pêndulo balístico utilizado para periciar projéteis de armamentos. Uma
perícia realizada em dois projéteis de massas iguais a 10 g pode indicar a arma utilizada em
um crime em que o primeiro projétil, após ser disparado contra o pêndulo, deslocou este de
uma altura h = 20 cm. Considerando que o segundo projétil deslocou o pêndulo de uma altura
h = 45 cm e que a massa do bloco é M = 4 kg, assinale a alternativa correspondente às veloci-
dades dos projéteis v1 e v2. Assuma a aceleração da gravidade igual a 10 m/s2.

O conteúdo deste livro eletrônico é licenciado para DÉBORA LIZ RIBEIRO STELLE - , vedada, por quaisquer meios e a qualquer título,
a sua reprodução, cópia, divulgação ou distribuição, sujeitando-se aos infratores à responsabilização civil e criminal.
www.grancursosonline.com.br 102 de 140
FÍSICA
Conservação da Quantidade de Movimento e Colisões
Herico Avohai

a) v1 = 1.203 m/s e v2 = 802 m/s.


b) v1 = 1.203 m/h e v2 = 802 m/h.
c) v1 = 802 cm/s e v2 = 1.203 cm/s.
d) v1 = 802 m/s e v2 = 802 m/s.
e) v1 = 802 m/s e v2 = 1.203 m/s

Letra e.
DADOS
mPROJÉTIL = 10 g = 0,01 kg
h1 = 20 cm = 0,2 m
Mbloco = 4 kg
Questão bem parecida com a que eu dei no exemplo sobre pêndulo balístico, vou até aprovei-
tá-la.
Pelas características do problema, podemos dizer que o choque (ou colisão) é classificado
como perfeitamente inelástico, logo não há conservação de energia durante o choque, entre-
tanto há conservação da quantidade de movimento.
Todavia, após o choque (depois que o projétil ficou alojado) podemos conservar a energia me-
cânica do sistema.
Portanto, primeiramente iremos calcular a velocidade que o conjunto (projétil + bloco) adquire
logo após a colisão e em seguida aplicar a conservação da Quantidade de Movimento para
encontrar a velocidade com que o projétil atingiu o bloco.

O conteúdo deste livro eletrônico é licenciado para DÉBORA LIZ RIBEIRO STELLE - , vedada, por quaisquer meios e a qualquer título,
a sua reprodução, cópia, divulgação ou distribuição, sujeitando-se aos infratores à responsabilização civil e criminal.
www.grancursosonline.com.br 103 de 140
FÍSICA
Conservação da Quantidade de Movimento e Colisões
Herico Avohai

Para encontrar a velocidade com que o conjunto (bloco + projétil) sai após o impacto, aplicare-

mos a Conservação da Energia Mecânica.

No ponto A, considerando o referencial nesse nível, o conjunto possui Energia Cinética diferen-

te de zero e Energia Potencial Gravitacional igual a zero.

No ponto B, o conjunto possui Energia Cinética igual a zero, pois atinge a altura máxima, e Ener-

gia Potencial Gravitacional diferente de zero.

Igualando as Energias mecânicas e substituindo os valores conhecidos,

O conteúdo deste livro eletrônico é licenciado para DÉBORA LIZ RIBEIRO STELLE - , vedada, por quaisquer meios e a qualquer título,
a sua reprodução, cópia, divulgação ou distribuição, sujeitando-se aos infratores à responsabilização civil e criminal.
www.grancursosonline.com.br 104 de 140
FÍSICA
Conservação da Quantidade de Movimento e Colisões
Herico Avohai

Para calcular a velocidade do projétil antes do choque, vamos aplicar a conservação da quan-
tidade de movimento:

ANTES DA COLISÃO
O vetor quantidade de movimento total antes será a soma dos vetores quantidades de movi-
mento do projétil e do bloco.

E o seu módulo será,

DEPOIS DA COLISÃO
O vetor quantidade de movimento total depois será a quantidade de movimento do conjunto
(projétil + bloco).

E o seu módulo será,

Substituindo na equação da conservação da quantidade de movimento, temos,

O conteúdo deste livro eletrônico é licenciado para DÉBORA LIZ RIBEIRO STELLE - , vedada, por quaisquer meios e a qualquer título,
a sua reprodução, cópia, divulgação ou distribuição, sujeitando-se aos infratores à responsabilização civil e criminal.
www.grancursosonline.com.br 105 de 140
FÍSICA
Conservação da Quantidade de Movimento e Colisões
Herico Avohai

Faça o mesmo para a altura 45 cm e encontrará vp = 1203 m/s. Qualquer dúvida pode entrar
em contato.

Questão 23 (MARINHA/OFICIAL/2015) Um projétil de massa m = 20g é atirado horizontal-


mente com velocidade v0 contra um pêndulo vertical cuja massa pendular é M = 2Kg, de fácil
penetração. O projétil aloja-se no pêndulo e, devido ao choque, o conjunto sobe até a altura h =
20cm. Determine a velocidade inicial do projétil e assinale a opção correta.
Dado: g= 10m/s2
a) 200m/s
b) 202m/s
c) 204m/s
d) 206m/s

Letra b.
DADOS
mPROJÉTIL = 20 g = 0,02 kg
h1 = 20 cm = 0,2 m
Mpendular = 2 kg
As questões de pêndulo balístico são muito parecidas. Podemos aproveitar o que fizemos nas
questões anteriores.
Pelas características do problema, podemos dizer que o choque (ou colisão) é classificado
como perfeitamente inelástico, logo não há conservação de energia durante o choque, entre-
tanto há conservação da quantidade de movimento.
Todavia, após o choque (depois que o projétil ficou alojado) podemos conservar a energia me-
cânica do sistema.
Portanto, primeiramente iremos calcular a velocidade que o conjunto (projétil + bloco) adquire
logo após a colisão e em seguida aplicar a conservação da Quantidade de Movimento para
encontrar a velocidade com que o projétil atingiu o bloco.

O conteúdo deste livro eletrônico é licenciado para DÉBORA LIZ RIBEIRO STELLE - , vedada, por quaisquer meios e a qualquer título,
a sua reprodução, cópia, divulgação ou distribuição, sujeitando-se aos infratores à responsabilização civil e criminal.
www.grancursosonline.com.br 106 de 140
FÍSICA
Conservação da Quantidade de Movimento e Colisões
Herico Avohai

Para encontrar a velocidade com que o conjunto (bloco + projétil) sai após o impacto, aplicare-
mos a Conservação da Energia Mecânica.

No ponto A, considerando o referencial nesse nível, o conjunto possui Energia Cinética diferen-
te de zero e Energia Potencial Gravitacional igual a zero.

No ponto B, o conjunto possui Energia Cinética igual a zero, pois atinge a altura máxima, e Ener-
gia Potencial Gravitacional diferente de zero.

Igualando as Energias mecânicas e substituindo os valores conhecidos,

O conteúdo deste livro eletrônico é licenciado para DÉBORA LIZ RIBEIRO STELLE - , vedada, por quaisquer meios e a qualquer título,
a sua reprodução, cópia, divulgação ou distribuição, sujeitando-se aos infratores à responsabilização civil e criminal.
www.grancursosonline.com.br 107 de 140
FÍSICA
Conservação da Quantidade de Movimento e Colisões
Herico Avohai

Para calcular a velocidade do projétil antes do choque, vamos aplicar a conservação da quan-
tidade de movimento:

ANTES DA COLISÃO
O vetor quantidade de movimento total antes será a soma dos vetores quantidades de movi-
mento do projétil e do bloco.

E o seu módulo será,

DEPOIS DA COLISÃO
O vetor quantidade de movimento total depois será a quantidade de movimento do conjunto
(projétil + bloco).

O conteúdo deste livro eletrônico é licenciado para DÉBORA LIZ RIBEIRO STELLE - , vedada, por quaisquer meios e a qualquer título,
a sua reprodução, cópia, divulgação ou distribuição, sujeitando-se aos infratores à responsabilização civil e criminal.
www.grancursosonline.com.br 108 de 140
FÍSICA
Conservação da Quantidade de Movimento e Colisões
Herico Avohai

E o seu módulo será,

Substituindo na equação da conservação da quantidade de movimento, temos,

Questão 24 (EXÉRCITO/ESPCEX/CADETE/2018) Dois fios inextensíveis, paralelos, idênti-

cos e de massas desprezíveis suspendem um bloco regular de massa 10 kg formando um

pêndulo vertical balístico, inicialmente em repouso. Um projetil de massa igual a 100 g, com

velocidade horizontal, penetra e se aloja no bloco e, devido ao choque, o conjunto se eleva a

uma altura de 80 cm, conforme figura abaixo. Considere que os fios permaneçam sempre pa-

ralelos. A velocidade do projetil imediatamente antes de entrar no bloco é

Dados: despreze a resistência do ar e considere a aceleração da gravidade igual a 10 m/s2.

O conteúdo deste livro eletrônico é licenciado para DÉBORA LIZ RIBEIRO STELLE - , vedada, por quaisquer meios e a qualquer título,
a sua reprodução, cópia, divulgação ou distribuição, sujeitando-se aos infratores à responsabilização civil e criminal.
www.grancursosonline.com.br 109 de 140
FÍSICA
Conservação da Quantidade de Movimento e Colisões
Herico Avohai

a) 224 m/s.
b) 320 m/s.
c) 370 m/s.
d) 380 m/s.
e) 404 m/s.

Letra e.
DADOS
mPROJÉTIL = 100 g = 0,1 kg
h1 = 80 cm = 0,8 m
MBLOCO = 10 kg
Pelas características do problema, podemos dizer que o choque (ou colisão) é classificado
como perfeitamente inelástico, logo não há conservação de energia durante o choque, entre-
tanto há conservação da quantidade de movimento.
Todavia, após o choque (depois que o projétil ficou alojado) podemos conservar a energia me-
cânica do sistema.
Portanto, primeiramente iremos calcular a velocidade que o conjunto (projétil + bloco) adquire
logo após a colisão e em seguida aplicar a conservação da Quantidade de Movimento para
encontrar a velocidade com que o projétil atingiu o bloco.

O conteúdo deste livro eletrônico é licenciado para DÉBORA LIZ RIBEIRO STELLE - , vedada, por quaisquer meios e a qualquer título,
a sua reprodução, cópia, divulgação ou distribuição, sujeitando-se aos infratores à responsabilização civil e criminal.
www.grancursosonline.com.br 110 de 140
FÍSICA
Conservação da Quantidade de Movimento e Colisões
Herico Avohai

Para encontrar a velocidade com que o conjunto (bloco + projétil) sai após o impacto, aplicare-
mos a Conservação da Energia Mecânica.

No ponto A, considerando o referencial nesse nível, o conjunto possui Energia Cinética diferen-

te de zero e Energia Potencial Gravitacional igual a zero.

No ponto B, o conjunto possui Energia Cinética igual a zero, pois atinge a altura máxima, e Ener-

gia Potencial Gravitacional diferente de zero.

O conteúdo deste livro eletrônico é licenciado para DÉBORA LIZ RIBEIRO STELLE - , vedada, por quaisquer meios e a qualquer título,
a sua reprodução, cópia, divulgação ou distribuição, sujeitando-se aos infratores à responsabilização civil e criminal.
www.grancursosonline.com.br 111 de 140
FÍSICA
Conservação da Quantidade de Movimento e Colisões
Herico Avohai

Igualando as Energias mecânicas e substituindo os valores conhecidos,

Para calcular a velocidade do projétil antes do choque, vamos aplicar a conservação da quan-

tidade de movimento:

ANTES DA COLISÃO
O vetor quantidade de movimento total antes será a soma dos vetores quantidades de movi-
mento do projétil e do bloco.

E o seu módulo será,

O conteúdo deste livro eletrônico é licenciado para DÉBORA LIZ RIBEIRO STELLE - , vedada, por quaisquer meios e a qualquer título,
a sua reprodução, cópia, divulgação ou distribuição, sujeitando-se aos infratores à responsabilização civil e criminal.
www.grancursosonline.com.br 112 de 140
FÍSICA
Conservação da Quantidade de Movimento e Colisões
Herico Avohai

DEPOIS DA COLISÃO
O vetor quantidade de movimento total depois será a quantidade de movimento do conjunto
(projétil + bloco).

E o seu módulo será,

Substituindo na equação da conservação da quantidade de movimento, temos,

Questão 25 (MARINHA/ESCOLA NAVAL/ASPIRANTE/2015) Analise a figura abaixo.

O conteúdo deste livro eletrônico é licenciado para DÉBORA LIZ RIBEIRO STELLE - , vedada, por quaisquer meios e a qualquer título,
a sua reprodução, cópia, divulgação ou distribuição, sujeitando-se aos infratores à responsabilização civil e criminal.
www.grancursosonline.com.br 113 de 140
FÍSICA
Conservação da Quantidade de Movimento e Colisões
Herico Avohai

A figura acima mostra o gráfico das energias cinéticas de dois carrinhos, A e B respectivamen-
te, que deslizam sem atrito ao longo de um trilho horizontal retilíneo. No instante t=3s ocorre
uma colisão entre os carrinhos. Sendo assim, assinale a opção que pode representar um gráfi-
co para as velocidades dos carrinhos antes e depois da colisão.
a)

b)

c)

O conteúdo deste livro eletrônico é licenciado para DÉBORA LIZ RIBEIRO STELLE - , vedada, por quaisquer meios e a qualquer título,
a sua reprodução, cópia, divulgação ou distribuição, sujeitando-se aos infratores à responsabilização civil e criminal.
www.grancursosonline.com.br 114 de 140
FÍSICA
Conservação da Quantidade de Movimento e Colisões
Herico Avohai

d)

e)

Letra a.
Observe o gráfico. O que acontece com a soma das energias antes e depois?
Antes da colisão a energia total do sistema é 14 J e depois também é 14J. Isso significa que a
colisão é perfeitamente elástica, pois a energia se conserva.
Para o corpo B.
Energia cinética antes,

O conteúdo deste livro eletrônico é licenciado para DÉBORA LIZ RIBEIRO STELLE - , vedada, por quaisquer meios e a qualquer título,
a sua reprodução, cópia, divulgação ou distribuição, sujeitando-se aos infratores à responsabilização civil e criminal.
www.grancursosonline.com.br 115 de 140
FÍSICA
Conservação da Quantidade de Movimento e Colisões
Herico Avohai

Energia cinética depois,

Igualando as equações I e II,

Ou seja, a velocidade inicial de B é 4 vezes maior que a sua velocidade final.


Por exclusão, a resposta é a letra a.

Questão 26 (MARINHA/ESCOLA NAVAL/ASPIRANTE/2011) Uma pista é composta por um


trecho retilíneo longo horizontal seguido do trecho circular vertical de raio R (conforme a figura
abaixo). O carrinho (1) (partícula), de massa m1 = 1,0 kg e velocidade = 5,0.î (m/ s), colide
com o carrinho (2) (partícula), de massa m2= 2,0kg, em repouso no trecho retilíneo. Despreze
os atritos. O coeficiente de restituição do choque vale 0,80. Após a colisão, o carrinho (2) sobe
o trecho circular vertical e, num certo instante, passa pela primeira vez na posição A, de altura
ha = R, com velocidade tal que o módulo da força normal da pista sobre o carrinho é igual ao
módulo do seu peso. Nesse instante, o módulo da velocidade (em m/ s) do carrinho (2) em
relação ao carrinho (1) é

O conteúdo deste livro eletrônico é licenciado para DÉBORA LIZ RIBEIRO STELLE - , vedada, por quaisquer meios e a qualquer título,
a sua reprodução, cópia, divulgação ou distribuição, sujeitando-se aos infratores à responsabilização civil e criminal.
www.grancursosonline.com.br 116 de 140
FÍSICA
Conservação da Quantidade de Movimento e Colisões
Herico Avohai

a) 1,0
b) 1,2
c) 2,5
d) 2,0
e) 3,0

Letra d.
DADOS
m1 = 1,0 kg

m2 = 2,0 kg
v2 = 0
e = 0,80
ha = R
Fn = P (ponto A)
v21 =?
Primeiramente vamos calcular as velocidades dos carrinhos após a colisão, para isso, utiliza-
remos a conservação a quantidade de movimento.

ANTES DA COLISÃO
O vetor quantidade de movimento total antes será a soma dos vetores quantidades de movi-
mento dos carrinhos, porém somente o carrinho 1 se movimenta.

O conteúdo deste livro eletrônico é licenciado para DÉBORA LIZ RIBEIRO STELLE - , vedada, por quaisquer meios e a qualquer título,
a sua reprodução, cópia, divulgação ou distribuição, sujeitando-se aos infratores à responsabilização civil e criminal.
www.grancursosonline.com.br 117 de 140
FÍSICA
Conservação da Quantidade de Movimento e Colisões
Herico Avohai

E o seu módulo será,

DEPOIS DA COLISÃO
O vetor quantidade de movimento total antes será a soma dos vetores quantidades de movi-
mento dos carrinhos, com os dois se movimentando para a direita.

E o seu módulo será,

Substituindo na equação da conservação da quantidade de movimento, temos,

O coeficiente de restituição é dado por,

O conteúdo deste livro eletrônico é licenciado para DÉBORA LIZ RIBEIRO STELLE - , vedada, por quaisquer meios e a qualquer título,
a sua reprodução, cópia, divulgação ou distribuição, sujeitando-se aos infratores à responsabilização civil e criminal.
www.grancursosonline.com.br 118 de 140
FÍSICA
Conservação da Quantidade de Movimento e Colisões
Herico Avohai

Lembre-se que é velocidade relativa, portanto,


A velocidade relativa de afastamento entre os carrinhos será igual a

A velocidade relativa de aproximação será,

Substituindo,

Resolvendo as duas equações,

E,

O conteúdo deste livro eletrônico é licenciado para DÉBORA LIZ RIBEIRO STELLE - , vedada, por quaisquer meios e a qualquer título,
a sua reprodução, cópia, divulgação ou distribuição, sujeitando-se aos infratores à responsabilização civil e criminal.
www.grancursosonline.com.br 119 de 140
FÍSICA
Conservação da Quantidade de Movimento e Colisões
Herico Avohai

Observe que após a colisão o corpo 1 inverte o sentido, pois a velocidade é negativa.
Para encontrar a velocidade no ponto A, devemos igualar a força normal com a força centrípeta.

O enunciado diz que Fn = P,

Agora, pensando na conservação de energia da partícula 2 no ponto imediatamente após a


colisão e no ponto A.

No ponto imediatamente após a colisão, a partícula 2 só possui energia cinética, já no ponto A,

ela possui energias cinética e potencial gravitacional.

O conteúdo deste livro eletrônico é licenciado para DÉBORA LIZ RIBEIRO STELLE - , vedada, por quaisquer meios e a qualquer título,
a sua reprodução, cópia, divulgação ou distribuição, sujeitando-se aos infratores à responsabilização civil e criminal.
www.grancursosonline.com.br 120 de 140
FÍSICA
Conservação da Quantidade de Movimento e Colisões
Herico Avohai

Cancelando as massas e substituindo os valores conhecidos,

Agora é só calcular a velocidade relativa de 2 em relação a 1,

Você lembra como faz?

Fixa o referencial que no caso é o 1 e inverte o seu vetor.

E o módulo será por Pitágoras,

O conteúdo deste livro eletrônico é licenciado para DÉBORA LIZ RIBEIRO STELLE - , vedada, por quaisquer meios e a qualquer título,
a sua reprodução, cópia, divulgação ou distribuição, sujeitando-se aos infratores à responsabilização civil e criminal.
www.grancursosonline.com.br 121 de 140
FÍSICA
Conservação da Quantidade de Movimento e Colisões
Herico Avohai

Questão 27 (NUCEPE/PERITO CRIMINAL/PC-PI/2012) Um atirador próximo dispara com

uma espingarda de ar comprimido um pequeno chumbinho de borracha com massa igual a

0,002kg. O projétil desloca-se com velocidade aproximadamente horizontal igual a 50m/s e

atinge uma pequena partícula esférica de massa 0,03kg, inicialmente parada. A partícula esfé-

rica está posicionada na extremidade de uma corda de comprimento igual a L = 2,0 m, confor-

me pode ser visto na figura abaixo. Assumindo que a colisão entre o projétil e a partícula seja

perfeitamente elástica, determine qual deve ser a altura aproximada h adquirida pela partícula

após a colisão.

a) 2,00 metros
b) 1,95 metro
c) 1,65 metro
d) 1,50 metro
e) 1,25 metro

Letra b.
DADOS
mPROJÉTIL = 0,002 kg
vP = 50 m/s
mESFERA = 0,03 kg
L – = 2,0 m

O conteúdo deste livro eletrônico é licenciado para DÉBORA LIZ RIBEIRO STELLE - , vedada, por quaisquer meios e a qualquer título,
a sua reprodução, cópia, divulgação ou distribuição, sujeitando-se aos infratores à responsabilização civil e criminal.
www.grancursosonline.com.br 122 de 140
FÍSICA
Conservação da Quantidade de Movimento e Colisões
Herico Avohai

h = 30 cm = 0,3 m
Analisando as quantidades de movimento antes e depois da colisão.
ANTES

Somente o projétil se movimenta.


DEPOIS

Considerando o sistema isolado e o referencial positivo para direita.


Para a conservação da quantidade de movimento, temos,
ANTES

DEPOIS

Substituindo os valores na equação da conservação da Quantidade de Movimento,

O conteúdo deste livro eletrônico é licenciado para DÉBORA LIZ RIBEIRO STELLE - , vedada, por quaisquer meios e a qualquer título,
a sua reprodução, cópia, divulgação ou distribuição, sujeitando-se aos infratores à responsabilização civil e criminal.
www.grancursosonline.com.br 123 de 140
FÍSICA
Conservação da Quantidade de Movimento e Colisões
Herico Avohai

Substituindo os valores conhecidos,

O enunciado diz que a colisão é perfeitamente elástica, portanto, o coeficiente de restituição é


igual a 1.
Lembre-se que é velocidade relativa, portanto,
A velocidade relativa de afastamento entre o projétil e a esfera será igual a

A velocidade relativa de aproximação será,

Logo,

Isolando na equação II,

O conteúdo deste livro eletrônico é licenciado para DÉBORA LIZ RIBEIRO STELLE - , vedada, por quaisquer meios e a qualquer título,
a sua reprodução, cópia, divulgação ou distribuição, sujeitando-se aos infratores à responsabilização civil e criminal.
www.grancursosonline.com.br 124 de 140
FÍSICA
Conservação da Quantidade de Movimento e Colisões
Herico Avohai

Substituindo na equação I,

Sabemos que a velocidade para a conservação das energias mecânicas nos pontos mais alto
e mais baixo é dada por,

Substituindo na equação acima, temos,

Elevando os dois membros ao quadrado,

Questão 28 (AERONÁUTICA/ASPIRANTE/AFA/2011) De acordo com a figura abaixo, a par-


tícula A, ao ser abandonada de uma altura H, desce a rampa sem atritos ou resistência do ar
até sofrer uma colisão, perfeitamente elástica, com a partícula B que possui o dobro da massa

O conteúdo deste livro eletrônico é licenciado para DÉBORA LIZ RIBEIRO STELLE - , vedada, por quaisquer meios e a qualquer título,
a sua reprodução, cópia, divulgação ou distribuição, sujeitando-se aos infratores à responsabilização civil e criminal.
www.grancursosonline.com.br 125 de 140
FÍSICA
Conservação da Quantidade de Movimento e Colisões
Herico Avohai

de A e que se encontra inicialmente em repouso. Após essa colisão, B entra em movimento e


A retorna, subindo a rampa e atingindo uma altura igual a

a) H
b) H/2
c) H/3
d) H/9

Letra d.
DADOS
mB = 2.mA
altura = H
ANTES
A partícula A é abandonada de uma altura H
VA0 = 0
vA =? (antes de colidir com B)
DEPOIS
B se movimenta
A retorna
H’A =?
Analisando as quantidades de movimento antes e depois da colisão.
ANTES

O conteúdo deste livro eletrônico é licenciado para DÉBORA LIZ RIBEIRO STELLE - , vedada, por quaisquer meios e a qualquer título,
a sua reprodução, cópia, divulgação ou distribuição, sujeitando-se aos infratores à responsabilização civil e criminal.
www.grancursosonline.com.br 126 de 140
FÍSICA
Conservação da Quantidade de Movimento e Colisões
Herico Avohai

Somente a partícula A se movimenta.

DEPOIS

Considerando o sistema isolado e o referencial positivo para direita.


- Para a conservação da quantidade de movimento.
ANTES

DEPOIS

Substituindo os valores na equação da conservação da Quantidade de Movimento,

Cortando os “mA”,

O conteúdo deste livro eletrônico é licenciado para DÉBORA LIZ RIBEIRO STELLE - , vedada, por quaisquer meios e a qualquer título,
a sua reprodução, cópia, divulgação ou distribuição, sujeitando-se aos infratores à responsabilização civil e criminal.
www.grancursosonline.com.br 127 de 140
FÍSICA
Conservação da Quantidade de Movimento e Colisões
Herico Avohai

O enunciado diz que a colisão é perfeitamente elástica, portanto, o coeficiente de restituição é


igual a 1.
Lembre-se que é velocidade relativa, portanto,
A velocidade relativa de afastamento entre A e B será igual a

A velocidade relativa de aproximação será,

Logo,

Isolando na equação II,

Substituindo na equação I,

O conteúdo deste livro eletrônico é licenciado para DÉBORA LIZ RIBEIRO STELLE - , vedada, por quaisquer meios e a qualquer título,
a sua reprodução, cópia, divulgação ou distribuição, sujeitando-se aos infratores à responsabilização civil e criminal.
www.grancursosonline.com.br 128 de 140
FÍSICA
Conservação da Quantidade de Movimento e Colisões
Herico Avohai

Sabemos que a velocidade para a conservação das energias mecânicas nos pontos mais alto
e mais baixo é dada por,

Substituindo na relação entre as velocidades de A antes e depois, temos,

Elevando os dois membros ao quadrado,

Questão 29 (MARINHA/OFICIAL/EFOMM/2016) Dois móveis P e T com massas de 15,0 kg


e 13,0 kg, respectivamente, movem-se em sentidos opostos com velocidades VP = 5,0 m/s e
VT = 3,0 m/s, até sofrerem uma colisão unidimensional, parcialmente elástica de coeficiente
de restituição e = 3/4. Determine a intensidade de suas velocidades após o choque.
a) VT = 5 m/s e VP = 3,0 m/s
b) VT = 4,5 m/s e VP = 1,5 m/s
c) VT = 3,0 m/s e VP = 1,5 m/s
d) VT = 1,5 m/s e VP = 4,5 m/s
e) VT = 1,5 m/s e VP = 3,0 m/s

Letra b.
DADOS
mP = 15 kg

O conteúdo deste livro eletrônico é licenciado para DÉBORA LIZ RIBEIRO STELLE - , vedada, por quaisquer meios e a qualquer título,
a sua reprodução, cópia, divulgação ou distribuição, sujeitando-se aos infratores à responsabilização civil e criminal.
www.grancursosonline.com.br 129 de 140
FÍSICA
Conservação da Quantidade de Movimento e Colisões
Herico Avohai

mT = 13 kg
vP = 5 m/s
vT = 3 m/s
e = 3/4
Conservando a quantidade de movimento,

ANTES DA COLISÃO
Os dois corpos movimentam-se em sentidos opostos,

DEPOIS DA COLISÃO
Os dois corpos afastam-se,

Temos que,

Substituindo os valores conhecidos,

O conteúdo deste livro eletrônico é licenciado para DÉBORA LIZ RIBEIRO STELLE - , vedada, por quaisquer meios e a qualquer título,
a sua reprodução, cópia, divulgação ou distribuição, sujeitando-se aos infratores à responsabilização civil e criminal.
www.grancursosonline.com.br 130 de 140
FÍSICA
Conservação da Quantidade de Movimento e Colisões
Herico Avohai

Sabemos que o coeficiente de restituição é dado por,

Resolvendo o sistema das equações I e II,

Multiplicando a equação II por 15,

Somando as equações,

E,

Questão 30 (IFB/PROFESSOR/IFB/2017) No alto de uma residência, apoia-se uma rampa


lisa na forma de um quadrante de circunferência de raio 0,45m. Do ponto A da rampa, abando-
na-se uma partícula de massa M que vai chocar-se elasticamente com outra partícula de mas-
sa 2M em repouso no ponto B, mais baixo da rampa. Considere g=10m/s2 e despreze todas

O conteúdo deste livro eletrônico é licenciado para DÉBORA LIZ RIBEIRO STELLE - , vedada, por quaisquer meios e a qualquer título,
a sua reprodução, cópia, divulgação ou distribuição, sujeitando-se aos infratores à responsabilização civil e criminal.
www.grancursosonline.com.br 131 de 140
FÍSICA
Conservação da Quantidade de Movimento e Colisões
Herico Avohai

as formas de atrito. Determine os módulos das velocidades em m/s, com que as partículas
chegam ao solo.

a) 1 e 2
b) 2 e 3
c) 3 e 4
d) 4 e 5
e) 5 e 6

Letra a.
DADOS
R = 0,45m
m1 = M
m2 = 2M
Qual a distância entre os pontos de impacto das partículas com o solo?
A partícula 1 descerá a rampa, colidirá com a partícula 2, em seguida, as duas são lançadas
horizontalmente percorrendo uma trajetória semiparabólica e caem no chão.
Partícula 1 é solta

O conteúdo deste livro eletrônico é licenciado para DÉBORA LIZ RIBEIRO STELLE - , vedada, por quaisquer meios e a qualquer título,
a sua reprodução, cópia, divulgação ou distribuição, sujeitando-se aos infratores à responsabilização civil e criminal.
www.grancursosonline.com.br 132 de 140
FÍSICA
Conservação da Quantidade de Movimento e Colisões
Herico Avohai

Colisão entre os corpos

Distância após o contato com o solo,

1º Encontrar a velocidade com que a partícula 1 atinge a partícula 2.


Utilizando a Lei da Conservação da Energia Mecânica para partícula 1,

No ponto A, a partícula 1 possui somente Energia Potencial Gravitacional e no ponto B, somen-


te Energia Cinética (referencial na base da rampa).

O conteúdo deste livro eletrônico é licenciado para DÉBORA LIZ RIBEIRO STELLE - , vedada, por quaisquer meios e a qualquer título,
a sua reprodução, cópia, divulgação ou distribuição, sujeitando-se aos infratores à responsabilização civil e criminal.
www.grancursosonline.com.br 133 de 140
FÍSICA
Conservação da Quantidade de Movimento e Colisões
Herico Avohai

2º Encontrar as velocidades das partículas após a colisão.


Aplicando a conservação da Quantidade de movimento,

Antes
Quem se movimenta é somente a partícula 1, logo,

Depois

Substituindo na equação da conservação da Quantidade de Movimento,

Colocando o “M” em evidência,

O conteúdo deste livro eletrônico é licenciado para DÉBORA LIZ RIBEIRO STELLE - , vedada, por quaisquer meios e a qualquer título,
a sua reprodução, cópia, divulgação ou distribuição, sujeitando-se aos infratores à responsabilização civil e criminal.
www.grancursosonline.com.br 134 de 140
FÍSICA
Conservação da Quantidade de Movimento e Colisões
Herico Avohai

Como a colisão é elástica, temos que o coeficiente de restituição é igual a 1, logo,

Resolvendo o sistema,

Somando as equações,

E,

Questão 31 (MARINHA/ESCOLA NAVAL/ASPIRANTE/2010) Dois pêndulos constituídos

por fios de massas desprezíveis e de comprimento L = 2,0 m estão pendurados em um teto em

dois pontos próximos de tal modo que as esferas A e B, de raios desprezíveis, estejam muito

próximas, sem se tocarem. As massas das esferas valem mA = 0,10 kg e mB = 0,15 kg. Aban-

dona-se a esfera A quando o fio forma um ângulo de 60º com a vertical, estando a esfera B do

outro pêndulo na posição de equilíbrio. Sabendo que, após a colisão frontal, a altura máxima

O conteúdo deste livro eletrônico é licenciado para DÉBORA LIZ RIBEIRO STELLE - , vedada, por quaisquer meios e a qualquer título,
a sua reprodução, cópia, divulgação ou distribuição, sujeitando-se aos infratores à responsabilização civil e criminal.
www.grancursosonline.com.br 135 de 140
FÍSICA
Conservação da Quantidade de Movimento e Colisões
Herico Avohai

alcançada pelo centro de massa do sistema, em relação à posição de equilíbrio, é de 0,40 m, o

coeficiente de restituição da colisão é Dado: | g | = 10,0 m/ s 2

a) zero

b) 0,25

c) 0,50

d) 0,75

e) 1,00

Letra e.

DADOS

L – = 2,0m

mA = 0,10 kg

mB = 0,15 kg

h = 0,40 m (altura do centro de massa)

Se você calcular as energias mecânicas antes e depois da colisão, classificará o tipo de co-

lisão.

ANTES DA COLISÃO

A altura da esfera A será,

O conteúdo deste livro eletrônico é licenciado para DÉBORA LIZ RIBEIRO STELLE - , vedada, por quaisquer meios e a qualquer título,
a sua reprodução, cópia, divulgação ou distribuição, sujeitando-se aos infratores à responsabilização civil e criminal.
www.grancursosonline.com.br 136 de 140
FÍSICA
Conservação da Quantidade de Movimento e Colisões
Herico Avohai

Então altura também será igual a 1,0 m.


Logo a energia mecânica antes da colisão será,

Após a colisão o enunciado diz que o centro de massa subiu 0,40m, portanto, a energia mecâ-
nica ao atingir a altura de 0,40m é de,

Opa, as energias são iguais, significa que houve conservação de energia, logo a colisão é do
tipo perfeitamente elástica e o coeficiente de restituição é igual a 1,0.

Chegamos ao final de mais uma aula! Parabéns para você que me acompanhou até aqui!
Não deixe de mandar as dúvidas!
Beijos na testa e até a próxima!

Hérico Avohai

Graduado em Física pela UNB e pós-graduado em Criminalística. É professor de Física, Matemática, Ra-
ciocínio Lógico e Criminalística, tendo começado a lecionar em 2000, tanto para o nível médio quanto para
cursos preparatórios para concursos. Foi aprovado em diversos concursos. Desde 2010 é Perito Criminal
da Polícia Científica do Estado de Goiás, atuou na Força Nacional de Segurança Pública entre 2016 e 2018
e é analistas em perfis de manchas de sangue pela Associação Internacional de Analistas de Manchas de
Sangue (IABPA).

O conteúdo deste livro eletrônico é licenciado para DÉBORA LIZ RIBEIRO STELLE - , vedada, por quaisquer meios e a qualquer título,
a sua reprodução, cópia, divulgação ou distribuição, sujeitando-se aos infratores à responsabilização civil e criminal.
www.grancursosonline.com.br 137 de 140
O conteúdo deste livro eletrônico é licenciado para DÉBORA LIZ RIBEIRO STELLE - , vedada, por quaisquer meios e a qualquer título,
a sua reprodução, cópia, divulgação ou distribuição, sujeitando-se aos infratores à responsabilização civil e criminal.
www.grancursosonline.com.br 138 de 140
O conteúdo deste livro eletrônico é licenciado para DÉBORA LIZ RIBEIRO STELLE - , vedada, por quaisquer meios e a qualquer título,
a sua reprodução, cópia, divulgação ou distribuição, sujeitando-se aos infratores à responsabilização civil e criminal.
www.grancursosonline.com.br 139 de 140
O conteúdo deste livro eletrônico é licenciado para DÉBORA LIZ RIBEIRO STELLE - , vedada, por quaisquer meios e a qualquer título,
a sua reprodução, cópia, divulgação ou distribuição, sujeitando-se aos infratores à responsabilização civil e criminal.

Você também pode gostar